GI pathology- QBANK AMBOSS

अब Quizwiz के साथ अपने होमवर्क और परीक्षाओं को एस करें!

52 years man -Nausea and bilious vomiting (after surgery); alleviated by prone position -Weight loss (bariatric surgery); abdominal distention -Succession splash (sloshing sound on abdominal auscultation) -No passage of contrast past the third segment of duodenum -Obstruction caused by? a) superior mesenteric artery b) body of pancreas c) gallbladder d) common bile duct e) portal vein f) inferior vena cava

-SMA passes anterior to 3rd segment of duodenum (transverse duodenum), and the aorta is located directly posterior to this part of intestines -Compression of duodenum by SMA= SMA syndrome -Classic presentation= postprandial pain, bilious vomiting following extreme weight loss option B= HEAD of pancreas can cause obstruction of distal 2nd and proximal 3rd part of duodenum; due to tumor or severe pancreatitis option D= cholangiocarcinoma of CBD near duodenum can cause obstruction of 2nd part of duodenum; wouldn't cause bilious vomiting, because bile release would be reduced

38 years woman -Fatigue and pruritus; hepatomegaly -Elevated ALP; serum antimitochondrial antibody titers elevated -Biopsy? a)fibrous, concentric obliteration of small and large bile ducts b)Intracytoplasmic eosinophilic inclusions in hepatocytes and cellular swelling c)Macrovesicular fatty infiltration and necrosis of hepatocytes d)Ballooning degeneration and apoptosis of hepatocytes e)Lymphocytic infiltration of portal areas and periductal granulomas

answer: Lymphocytic infiltration of portal areas and periductal granulomas -Cholestatic signs + elevated antimitochondrial antibodies + woman= PBC -Lymphocytic inflammation of portal area with periductal granulomatous changes and eventual bile duct destruction option A= PSC option B= alcoholic hepatitis option C= steatohepatitis option D= acute viral hepatitis or steatohepatitis

63 years man -Fatigue, crampy abdominal pain, watery diarrhea, pain in his mouth and gums (4 weeks); traveled to Dominican (2 weeks) -Smoker (45 years) -Three 1.5 cm painful ulcers in mouth; Mild tenderness in RLQ -Low hemoglobin, low MCV, high WBCs -Colonoscopy: Cobblestone mucosa -Biopsy: noncaseating granuloma; cause? a) Tropical sprue b) Behcet disease c) Crohn disease d) Whipple disease e) Ulcerative colitis

answer: Crohn disease -Bimodal distribution and can develop in older patients -Presentation: chronic, watery, nonbloody diarrhea, RLQ pain; oral aphthae -Lab: microcytic anemia and leukocytosis -Heavily associated with smoking -Biopsy= granulomatous inflammation -Colonoscopy: cobblestone mucosa, linear ulcers, transmural inflammation, fistulas and aphthous hemorrhagic mucosa option A= traveling to tropics (Dominican); usually have to stay more than one month; presents with malabsorption, steatorrhea, megaloblastic anemia (deficiency of folate and B12); biopsy= villous atrophy and elongated crypts option B= painful oral ulcers, abdominal pain and diarrhea; more common in Mediterranean region (eastern Asia, Turkey); genital ulcers too; lacks lesions of skin (erythema nodosum); eye (anterior uveitis, retinal vasculitis) or vasculature (phlebitis, DVT); granulomatous inflammation not expected option D= abdominal pain, bloating diarrhea, malabsorption; arthritis, sacroiliitis, cardiac and neurological symptoms; oral ulcers not expected; biopsy= PAS-positive macrophages (within small intestines)) option E= chronic bloody diarrhea and abdominal pain; neutrophilic crypt abscesses; no granuloma or oral ulcers; smoking is protective

46 years man -Black liver; history of intermittent scleral icterus -High direct bilirubin -Diagnosis? a)Type II Crigler-Najjar syndrome b)Dubin-Johnson syndrome c)Rotor syndrome d)Gilbert syndrome e)Type I Crigler-Najjar syndrome f)Wilson disease

answer: Dubin-Johnson syndrome -All signs together= Dubin-Johnson syndrome -Hereditary defect in transport protein of biliary canaliculi -Impairs conjugated bilirubin excretion and causes high direct bilirubin -Also impairs excretion of epinephrine metabolites= liver black -Benign condition and doesn't require treatment option A= decreased UDP-glucuronosyltranferase synthesis= indirect hyperbilirubinemia option C= high direct; doesn't cause pigmentation of liver option D= high indirect; mild decrease in UDP-glucuronosyltranferase activity; jaundice in stress only option E= absent UDP-glucuronosyltranferase; indirect is high; usually causes kernicterus and death

45 years woman -Recurrent retrosternal chest pain (worse at night) -Endoscopy= hyperemia in distal third of esophagus -Biopsy= nonkeratinized stratified squamous epithelium with hyperplasia of the basal cell layer and neutrophilic infiltrates -Underlying cause? a)Increased lower esophageal sphincter tone b)Increased collagen production and fibrosis c)Chronic gastrointestinal iron loss d)Dysfunction of the gastroesophageal junction e)Spread of neoplastic cells f)Metaplastic transformation of esophageal epithelium

answer: Dysfunction of the gastroesophageal junction -Clinical findings + histology= GERD -Dysfunction of GE junction allows stomach contents to flow back into esophagus, causing inflammation of the esophageal epithelium -Risk factors for GERD: dysfunction of LES due to anatomical disruption (e.g., hiatal hernia), decreased tension (e.g., smoking) and transient relaxation (e.g., obesity) option A= achalasia option B= CREST syndrome option C= Plummer-Vinson syndrome; triad of dysphagia, upper esophageal webs, and iron deficiency anemia option E= esophageal cancer option F= Barrett esophagus

51 years man -acute viral HBV (9 months ago) -Serum studies: increased hepatic transaminase activity and an HBV viral DNA load of 4286 IU/mL -Which set is most likely? a) HBeAg= negative, Anti-HBc= positive, Anti-HBc IgG= positive, Anti-HBc IgM= negative b) HBeAg= negative, Anti-HBc= negative, Anti-HBc IgG= positive, Anti-HBc IgM= negative c) HBeAg= negative, Anti-HBc= positive, Anti-HBc IgG= negative, Anti-HBc IgM= negative d) HBeAg= negative, Anti-HBc= negative, Anti-HBc IgG= negative, Anti-HBc IgM= positive e) HBeAg= positive, Anti-HBc= negative, Anti-HBc IgG= positive, Anti-HBc IgM= negative f) HBeAg= positive, Anti-HBc= negative, Anti-HBc IgG= negative, Anti-HBc IgM= positive

answer: E -Acute HBV + high transaminases activity and a viral DNA load >2000 IU/ml= active chronic hepatitis B -Chronic HBV= persistence of HBsAg and HBV DNA (more than 6 months after initial infection) and absence of anti-HBs -Anti-HBc IgG may be present or absent with chronic HBV -Increased HBeAg= active (if absent= inactive) option A= complete recovery from acute HBV infection option B= inactive chronic HBV; but also, normal transaminases and <2000IU/ml viral load option C= vaccination option D= window period of acute HBV; different symptoms option F= acute viral hepatitis B

15 months girl -Grossly bloody diarrhea (2 days); few episodes of blood-tinged stools (3 weeks) -No pain, no nausea or vomiting -55th percentile for weight and height -Abdomen soft and nontender; low Hb, low MCV -Further evaluation? a)Double bubble sign on abdominal X-ray b)Cobblestone mucosa on colonoscopy c)Neutrophil infiltrated crypts on colonic biopsy d)Absent ganglionic cells on rectal suction biopsy e)Target sign on abdominal ultrasound f)Ectopic gastric mucosa on Technetium-99m pertechnetate scan

answer: Ectopic gastric mucosa on Technetium-99m pertechnetate scan -Young girl + Painless lower GI bleeding + microcytic anemia= Meckel diverticulum (true diverticulum) -T-99m scan= shows ectopic gastric mucosa (or pancreatic) in small bowel= diagnostic of Meckel diverticulum in children -Acid-secretion by gastric mucosa within the diverticulum results in ileal ulceration and subsequent bleeding= iron-deficiency anemia option A= duodenal atresia; bilious vomiting option B= Crohn; pain, nonbloody diarrhea and older patients option C= UC; pain, tenesmus, extraintestinal manifestations (e.g., uveitis, arthritis, erythema nodosum); older patients option D= Hirschsprung disease, failure to pass meconium option E= intussusception; pain, vomiting and tenderness as well as bleeding

71 years woman -Fatigue (8 months) -Labs: high ALP; evaluation of helps determine cause? a)Cancer antigen 27-29 b)Lactate dehydrogenase c)Ferritin d)Gamma-glutamyl transpeptidase e)Calcitriol

answer: Gamma-glutamyl transpeptidase -High ALP, GGTP should be used to differentiate between disease of hepatobiliary system (both elevated) from a disease of bone (only ALP elevated) -Increased GGTP and normal ALP= heavy alcohol use option A= advanced breast cancer option B= nonspecific marker in malignancies, hemolytic anemia, hepatocyte injury and infarction= elevated option C= systemic inflammation, malignant disease, anemia of chronic disease, sideroblastic anemia= elevated option E= diagnose vitamin D deficiency; ALP can be elevated (due to secondary hyperparathyroidism; before looking at bone diseases; measure GGTP to see if bone is the problem

10 years girl -Bloody diarrhea and abdominal pain (2 days) -Visited petting zoo (4 days ago), fever, tenderness in RLQ -Stool cultures at 42C grow colonies that turn black after adding phenylenediamine -Cause? a) Gram-negative, flagellated bacteria that ferment lactose b) Gram-negative, non-flagellated bacteria that do not ferment lactose c) Gram-positive, anaerobic, rod-shaped bacteria that form spores d) Gram-negative, flagellated bacteria that do not ferment lactose e) Gram-positive, aerobic, rod-shaped bacteria that produce catalase f) Gram-positive, aerboic, rod-shaped bacteria that form spores

answer: Gram-negative, flagellated bacteria that do not ferment lactose -RLQ pain + bloody diarrhea, fever after exposure to animals + stool cultures that grow oxidase-positive organism (turn black with phenylenediamine) at 42C= Campylobacter species -C. jejuni: comma-shaped, non-lactose fermenting with polar flagella -Leading cause of bloody diarrhea in children -Transmission: fecal-orally (undercooked, contaminated poultry/meat, unpasteurized milk, fomites containing pathogen); direct contact with infected animals (dogs, cats, pigs) -Campylobacter infection can lead to Guillain-Barre syndrome and reactive arthritis -Salmonella: also gram-negative, flagellated, do not ferment lactose, fever, abdominal pain, bloody diarrhea in young children; exposure to reptile and live poultry= risk; but test negative for oxidase option A= Enterohemorrhagic E. coli, shiga-toxin producing; fever, bloody diarrhea, abdominal pain; Cows are common carriers; fecal-oral; negative for oxidase option B= Yersinia and shigella spp.; bloody diarrhea, abdominal pain, fever; Yersinia enterocolitis infections associated with animal exposure (deer, cattle, pigs); however, both are negative for oxidase option C= Clostridium spp.; C. perfringens= watery diarrhea, uncooked food; C. difficile= antibiotic use; C. botulinum= diarrhea with neurological dysfunctions; Clostridium spp. Are negative for oxidase option E= Listeria monocytogenes; fever, diarrhea (non-bloody), abdominal pain; negative for oxidase; acquired via ingestion of unpasteurized dairy option F= Bacillus cereus; copious diarrhea (non-bloody), fever, abdominal pain; negative for oxidase; ingestion of food left at room temperature (fried rice syndrome)

23 years woman -age: 23 years woman -Diarrhea, flatulence, fatigue (2 months) -3-5 episodes of loose stools daily (oily appearance) -Worse after eating -Itchy rash on her elbows and knees; dermatitis herpetiformis -Evaluation would show? a) macrocytic, hypochromic red blood cells b) low fecal elastase levels c) PAS-positive intestinal macrophages d) HLA-DQ2 serotype e) elevated exhaled hydrogen concentration f) elevated urine tryptophan levels

answer: HLA-DQ2 serotype -HLA-DQ2 in 90-95% of cases of Celiac disease --> predisposes patients due to increased binding of gluten peptides to this HLA subtype, triggering T-cell autoimmune reaction and inflammation of intestinal epithelium -Dermatitis herpetiformis (caused by IgA and C3 deposition in dermal papillae of the dermis) option A= malabsorptive conditions; vitamin B12 deficiency and/or folate deficiency; in tropical sprue, atrophic gastritis, chronic pancreatitis; Celiac disease= iron deficiency anemia; microcytic, hypochromic anemia option B= pancreatic exocrine insufficiency option C= Whipple disease option E= lactose intolerance or small intestinal bacterial overgrowth; lactose/hydrogen breath test option F= Hartnup disease; diarrhea and skin rash (niacin deficiency); rash involves neck (Casal necklace) and manifests with hyperpigmentation to sun exposed areas; also dementia, glossitis

21 years man -Was in Mexico (2 weeks ago) -Nausea, vomiting, fever, abdominal pain -Scleral icterus, RUQ tenderness; hepatomegaly -Biopsy? a)Concentric periductal fibrosis b)Ground glass hepatocytes and periportal inflammation c)Hepatocyte swelling and bridging necrosis d)Necrosis with neutrophilic infiltration and Mallory bodies e)Lymphocytic infiltration and progressive ductopenia f)Piecemeal necrosis and fatty changes

answer: Hepatocyte swelling and bridging necrosis -Signs of HAV as well recent travel to Mexico= only acute form -Ballooning degeneration (swelling of hepatocytes due to depletion of ATP) and bridging necrosis (spanning adjacent lobules)= acute viral hepatitis -Other findings: Councilman bodies and periportal infiltration with helper T cells, B cells, plasma cells option A= PSC option B= chronic HBV option D= alcoholic hepatitis option E= PBC option F= chronic HBV or HCV

43 years woman -age: 43 years woman -Alcohol daily (20 years) -Jaundice and hepatosplenomegaly -Biopsy: Mallory bodies and microvesicular steatosis -Mechanism? a)Excessive interstitial TGF-beta activity b)Decreased clearance of N-acetyl-p-benzoquinone imine c)Intracellular accumulation of lactate d)Increased glycerol 3-phosphate formation e)Estrogen-mediated glandular hyperplasia

answer: Increased glycerol 3-phosphate formation -Chronic alcoholism + hepatic injury + ballooning degeneration with Mallory body + microvesicular steatosis= steatohepatitis -Increased G3P formation from DHAP (dihydroxyacetone phosphate) tends to occur when there is high concentration of NADH, which is present in patients who drink -Massive hepatic degradation of ethanol to acetyl-CoA causes NADH excess; increase in both G3P and availability of fatty acids causes increased triglyceride synthesis, leading to development of steatohepatitis option A= associated with hepatocyte necrosis (not this stage), involved in cirrhosis option B= acetaminophen toxicity option C= and lactate acidosis occurs with increased activity of alcohol dehydrogenase, which breaks down ethanol by reducing NAD to NADH; increased NADH favors conversion of pyruvate to lactate in an attempt to recycle NAD+; not mechanism underlying steatohepatitis directly option E= hepatocellular adenoma

newborn female -Premature, vomiting and becomes inconsolable 48 hours after birth -Hasn't passed meconium; abdominal distention, high pitched bowel sounds -Contrast enema= microcolon; serology= increased levels of immunoreactive trypsinogen -Additional findings? a)Decreased hydrogen ion concentration in the renal collecting duct b)Increased chloride concentration in alveolar fluid c)Increased serum calcium concentration d)Increased bicarbonate concentration in pancreatic secretions e)Increased sodium concentration in sweat

answer: Increased sodium concentration in sweat - cystic fibrosis -Most common CFTR gene mutation= membrane transporter misfolds= retained in the RER= chloride is not properly reabsorbed and remains in the lumen of apocrine sweat glands -Positively charged sodium ions are also trapped in sweat by negatively charged chloride ions in order to balance net electric charge= hypertonic sweat option A= vomiting= hypovolemic= contraction alkalosis (low HCO3- concentration= RAAS increases reabsorption of HCO3- and Na and excretes H+ and K)= CFTR not expressed in kidneys option B= no CFTR= no transport of chloride into alveoli= water not secreted and resulting mucus is hyperviscous= mucus plug formation option C= in severe CF; fat malabsorption= less vitamin D= less intestinal and renal calcium reabsorption option D= in normal eating; CF impairs secretion of chloride, sodium and bicarbonate ions from pancreas= decreased water content= development of recalcitrant meconium; hyperviscous pancreatic secretions= sclerosis of pancreatic ducts= exocrine pancreatic insufficiency

55 years man -Chronic HBV; fatigue, weight loss (4 months) -Hepatomegaly; high alpha-fetoprotein; CT= solitary mass in left lobe of liver -Underlying pathogenesis? a)Overexpression of secretory hepatitis antigen b)Intracellular accumulation of misfolded protein c)Gain of function mutation of a proto-oncogene d)Viral cytotoxin-induced cellular dysplasia e)Integration of foreign DNA into host genome

answer: Integration of foreign DNA into host genome -HCC -Integration of HBV DNA into host genome= alters expression of endogenous host genes (important for cell growth, proliferation and differentiation)= also, induces chromosomal instability -Viral DNA integration= random= usually involves segments encoding for key factors in carcinogenesis (p53, pRb, cyclins A and D1, TGF-beta) -Continuous expression of certain viral proteins (e.g., HBx protein) modulate expression of host genes= implicated in HCC option A= active replication and high infectivity; in acute HBV and alone doesn't induce carcinogenesis option B= AATD option C= contributes to carcinogenesis in general; no specific proto-oncogene associated with HCC option D= HBV doesn't have direct cytotoxic effect; but infected hepatocytes are destroyed by CD8+ T cells; chronic intrahepatic inflammation maintains a cycle of liver cell necrosis, mitosis, regenerative hyperplasia= cellular dysplasia= HCC

24 years man -Fever, crampy abdominal pain, and bloody diarrhea (2 days) -Returned from Mexico; diffuse tenderness to palpation; bowel sounds= hyperactive -Stool cultures= non-lactose fermenting, oxidase negative, gram negative rods; do not produce hydrogen sulfide on triple sugar iron agar -Pathogenesis? a)Invasion of colonic microfold cells b)Inhibition of host cytoskeleton organization c)Overactivation of adenylate cyclase d)Dissemination via bloodstream e)Flagella-mediated gut colonization

answer: Invasion of colonic microfold cells -Shigella -Shigella enters cells (invades colonic microfold cells) via pinocytosis before lysing vesicles and entering the cytoplasm -Inside cytoplasm, shigella produces Shiga toxin and rapidly poisons cell -Key to Shigella's ability to cause disease even in cases of very small exposure to bacteria (e.g., Shigella has very low infectious dose) -Once in colonic mucosa, shigella travels from cell to cell via actin polymerization option B= via actin depolymerization= mechanism of action of toxin B, secreted by C. difficile; gram-positive rod associated with antibiotic use option C= Cholera toxin and of heat-labile toxin (enterotoxigenic E. coli)= increase cAMP, leading to higher chloride secretion with resulting water efflux= non-bloody diarrhea; V. cholerae= oxidase positive and comma-shaped; E.coli= rod-shaped by fast lactose fermenter option D= Salmonella virulence; contracted after eating poultry or eggs and manifests with bloody diarrhea; H2S producer when grown on TSI agar option E= C. jejuni virulence; bloody diarrhea; high fever, myalgias, malaise; Oxidase positive

62 years man -Fatigue, dyspnea, straining during defecation, weight loss -Conjunctival pallor; microcytic anemia -Stool occult positive -Colonoscopy: exophytic mass in ascending colon= well-differentiated adenocarcinoma -Gain of function mutation in? a) DCC b) TP53 c) MLH1 d) APC e) KRAS

answer: KRAS -sporadic= chromosomal instability pathway (adenoma-carcinoma sequence) -1) Loss of function in APC (tumor suppressor)= decreased intercellular adhesion and increased epithelial proliferation -2) Gain of function mutation in KRAS (proto-oncogene)= unregulated cell signaling and proliferation= adenoma formation -3) loss of mutations in tumor suppressors (TP53, DCC)= malignant transformation of adenoma to carcinoma option B= late loss of function mutation; normally, TP53 is activated in response to DNA damage and inhibits G1-S progression and allows for DNA repair; TP53 can also activate apoptosis if DNA damage is irreparable; associated with most human cancers and Li-Fraumeni syndrome option C= microsatellite instability pathway= loss of function mutation or epigenic silencing (promoter hypermethylation) of mismatch repair genes, including MLH1= colorectal carcinoma (Lynch syndrome, not sporadic) option D= sporadic and familial adenomatous polyposis

55 years woman -Painful swelling (firm and tender) on right side of face (worse when eating) -US= stone located in a duct that runs anterior to masseter muscle and passes through buccinator muscle -Sialoendoscopy to remove stone; which site must be inserted? a)Lateral to the superior labial frenulum b)Lateral to the lingual frenulum c)Into the foramen cecum d)Lateral to the second upper molar tooth e)Into the floor of the mouth f)Into the mandibular foramen

answer: Lateral to the second upper molar tooth -Affects parotid duct -Anterior edge of parotid gland= turns medially at anterior edge of masseter= pierces through buccinator= enters oral cavity lateral to the 2nd upper molar -Sialolithiasis affects parotid gland or duct in 20% of cases -risk factors: dehydration, certain medications (e.g., anticholinergics), trauma -Treated with NSAIDs and stimulation of salivary flow by gland massage or warm compresses -Severe cases= Sialoendoscopy or open surgical procedures option A and F= no salivary ducts option B= submandibular duct; from deep part of submandibular gland and opens into oral cavity lateral to lingual frenulum; 80% of stones in submandibular gland or duct option C= small, midline indentation in the posterior aspect of tongue; embryological remnant of thyroglossal duct; if duct fails to obliterate= midline neck cysts or fistulas can develop; no salivary gland drains into it option E= sublingual gland is drained by numerous glands; largest one joins submandibular duct, while remaining small ducts drain into floor of mouth

64 years woman -Nausea, vomiting (2 days); past week= acetaminophen frequently for knee pain -Scleral icterus, tender hepatomegaly -Confused and high AST, ALT -Underlying mechanism? a)glucuronide-conjugate formation b)Salicylic acid formation c)N-acetyl-p-benzoquinoneimine formation d)N-acetylcysteine formation e)Sulfate-conjugate formation

answer: N-acetyl-p-benzoquinoneimine formation -Acetaminophen toxicity -N-acetyl-p-benzoquinoneimine formation is a toxic breakdown product of acetaminophen -at therapeutic doses= glutathione acts to inhibit toxic NAPQI -In overdose= glutathione reserves are depleted, and concentrations of NAPQI build up, leading to acute liver failure option A= mechanism of acetaminophen clearance option B= metabolite of aspirin; could also cause liver failure option D= antidote used in management of acetaminophen induced liver failure, rather than cause of liver damage; helps restore glutathione levels and enhances inactivation of toxic metabolites of acetaminophen option E= mechanism of acetaminophen excretion

68 years man mid-esophagus biopsy: keratin pearls and intercellular bridging explanation? a)well-differentiated neoplastic glandular proliferation b)Atrophy and fibrosis of the esophageal smooth muscle c)Metaplastic transformation of esophageal mucosa d)Neoplastic proliferation of squamous epithelium e)Eosinophilic infiltration of esophageal walls

answer: Neoplastic proliferation of squamous epithelium - SCC of esophagus option A= adenocarcinoma option B= CREST syndrome option C= Barrett esophagus option E= eosinophilic esophagitis

37 years woman -Weight loss, bloating, diarrhea (6 months) -Bilateral white spots on the temporal half of the conjunctiva, dry skin and hard neck mass in the anterior midline that does not move with swallowing -Urinalysis after a D-xylose meal shows a normal increase in renal D-xylose excretion -Which could have prevented weight loss? a)Gluten-free diet b)Pancreatic enzyme replacement c)Tetracycline therapy d)Mesalamine therapy e)Lactose-free diet

answer: Pancreatic enzyme replacement -Xeroderma and keratin deposits on the conjunctivae (Bitot's spots)= vitamin A deficiency -Physiologic increase in D-xylose excretion seen in this patient indicates that intestinal mucosa is normal -Mass in anterior midline= Riedel's thyroiditis, which is associated with other IgG4-related systemic diseases -Vitamin A deficiency + IgG4 disease + normal D-xylose response= autoimmune pancreatitis (causing exocrine pancreatic insufficiency) option A= Celiac; D-xylose test would have shown decreased xylose excretion due to damage of intestinal mucosa option C= combination with folic acid= tropical sprue treatment; diarrhea, deficiency of fat-soluble vitamins; doesn't manifest with thyroid mass, and decreased xylose excretion; after traveling to tropics option D= ulcerative colitis; no neck mass or vitamin A deficiency; no Bitot's spots (but can show uveitis or episcleritis; normal D-xylose test option E= lactose intolerance; normal D-xylose; after consumption of dairy; not associated with fat soluble vitamin deficiencies or neck mass

43 years woman -Worsening heartburn, abdominal pain (4 months) -Multiple episodes of greasy diarrhea -6 months ago= similar symptoms= duodenal ulcers -Mother: died (uncontrolled hypoglycemia and had primary hyperparathyroidism) -Medications: pantoprazole, ranitidine -EGD: friable ulcer in distal duodenum -Further evaluation will show? a)anti-intrinsic factor antibodies in the serum b)Anti-tissue transglutaminase antibodies in the serum c)Parietal cell hyperplasia in the stomach d)Noncaseating granuloma in the jejunum e)Dystrophic calcifications in pancreas

answer: Parietal cell hyperplasia in the stomach -Treatment resistant ulcers + malabsorption + family history of MEN (primary hyperparathyroidism and pancreatic insulinoma)= Zollinger-Ellison syndrome -Parietal cell hyperplasia seen in ZES (gastrin secreting tumor)= gastric acid overproduction Neuroendocrine tumors carry somatostatin receptors, radiolabeled octreotide can be used diagnostically to locate tumor option A= pernicious anemia option B= celiac disease option D= crohn disease option E= chronic pancreatitis

28 years man -Progressive fatigue and intermittent diarrhea (6 months) -Weight loss; pale conjunctivae; colonoscopy: erythematous and inflamed mucosa with fibrin-covered ulcerations -Further evaluation? a)Positive lactose hydrogen breath test b)Anti-Saccharomyces cerevisiae antibodies c)Perinuclear antineutrophil cytoplasmic antibodies d)PAS-positive cytoplasmic granules e)Anti-tissue transglutaminase antibodies

answer: Perinuclear antineutrophil cytoplasmic antibodies signs suggest= ulcerative colitis -pANCA= antibodies against myeloperoxidase granules in perinuclear region of cytoplasm (80% in UC); no correlation between titer and disease activity -pANCA (also)= Churg-Strauss syndrome, microscopic polyangiitis and PSC option A= lactose intolerance; normal intestinal mucosa option B= ASCA in Crohn (up to 70%) option D= Whipple disease; endoscopy= pale yellow, eroded patches option E= Celiac disease, endoscopy= villous atrophy and crypt hyperplasia (a smooth or atrophic mucous membranes)

49 years man -Chest pain that radiates to her back (while eating) -Feeling of food, both liquid and solid, getting stuck in her chest while she is easting (past 3 months) -Esophagogram: corkscrew or rosary bead apperance -Further evaluation would show? a)Simultaneous multi-peak contractions on manometry b)Multiple mucosal erosions on endoscopy c)Elevated lower esophageal sphincter pressure on manometry d)Gastroesophageal junction mass on endoscopy e)Hypertensive contraction on manometry

answer: Simultaneous multi-peak contractions on manometry -Corkscrew or rosary bead appearance= diffuse esophageal spasm= simultaneous multi-peak contractions on manometry -Repetitive nonperistaltic, nonprogressive contractions impede the progression of solid and liquid foods down esophagus= dysphagia and/or squeezing retrosternal chest pain -Treatment: calcium channel blockers, anticholinergics, or nitrates option B= reflux esophagitis option C= achalasia; same presentation but esophagogram= bird-beak sign option D= gastric adenocarcinoma; esophagogram= apple-core appearance option E= hypertension peristalsis; esophagogram= nutcracker esophagus

21 years woman -Fatigue, intermittent abdominal pain, bulky, foul-smelling diarrhea, weight loss (2 months) -Sudan III stain (stool)= large number of red droplets -Cause? a) ulcerative colitis b) carcinoid syndrome c) amebiasis d) irritable bowel syndrome e) lactose intolerance f) celiac disease

answer: celiac disease -Celiac disease= hypersensitivity to gliadin -Duodenum and jejunum (proximal parts) -Villous atrophy, crypt hyperplasia, intraepithelial lymphocytosis -Often asymptomatic; crampy abdominal pain, fat malabsorption and steatorrhea -Symptoms of fat-soluble vitamin deficiency, anemia, depression, dermatitis herpetiformis -Positive IgA anti-tissue transglutaminase, anti-endomysial, and anti-deamidate gliadin peptide antibodies -Associated with HLA-DQ2 and HLA-DQ8 option A= bloody diarrhea option B= diarrhea but not steatorrhea option C= bloody diarrhea option D= fluctuating diarrhea/constipation; no steatorrhea option E= alone wouldn't cause fat malabsorption

57 years man -Chronic, retrosternal chest pain (worse after meals and night) -PPIs= no relief -Endoscopy: ulcerations in distal esophagus and proximally dislocated Z line -Biopsy (distal esophagus)= mature columnar epithelium with goblet cells -Same pathogenesis? a)Pseudostratified columnar epithelium in the bronchi b)Squamous epithelium in the bladder c)Paneth cells in the duodenum d)Branching muscularis mucosa in the jejunum e)Chief cells in the ileum f)Simple columnar epithelium in the endocervix

answer: Squamous epithelium in the bladder Barrett esophagus -Bladder= normally transitional epithelium -Chronic irritation of bladder wall (e.g., from Schistosoma infection, urinary canaliculi, indwelling catheters)= transformation to squamous epithelium -Cellular transformation due to chronic stress (metaplasia) occurs when stem cells reprogram to replace one mature cell type to another (that adapts with chronic stress) -Metaplasia (reversible)= if chronic irritation persists= dysplasia option A= normal; stratified squamous epithelium in bronchi due to chronic smoking= metaplasia option C= normal; Paneth cells in stomach= metaplasia) option D= hamartomous polyps; associated with Peutz-Jeghers syndrome; can occur anywhere in GI option E= gastric ectopic tissue (Choristoma)= Meckel diverticulum; metaplasia but during embryogenic development; Barrett esophagus= postnatal metaplasia option F= normal; transform into squamous epithelium= metaplasia (due to exposure to acid vaginal pH)

45 years woman finding to support lactose intolerance? a)Partial villous atrophy with eosinophilic infiltrate b)Periodic acid-Schiff positive foamy macrophages c)Crypt abscesses and colonic ulcerations d)Tall villi with focal collections of goblet cells e)Duodenal epithelium with dense staining for chromogranin A f)Villous atrophy and crypt hyperplasia g)Noncaseating granulomas with lymphoid aggregates

answer: Tall villi with focal collections of goblet cells -Tall villi lined by simple columnar epithelium and crypts with interspaced goblet cells and Paneth cells= normal jejunum -Deficient in lactase, permitting osmotically active lactose to pass undigested to large bowel= binds water and acts as substrate for colonic bacteria= flatulence, bloating and abdominal pain option A= Cow's milk allergy; abdominal pain and bloating, diarrhea (in infants); diagnosis= oral milk challenge with identification of serum cow milk-specific IgE option B= Whipple disease option C= ulcerative colitis option E= in subset with irritable bowel syndrome; episodic abdominal cramps, bloating and flatulence; changes in stool frequency and form (alternating diarrhea and constipation) option F= celiac disease option G= Crohn disease

70 years man -Constipation (1 year); sometimes uses fingers for evacuation -Defecation is painful and preceded by sense of bowel obstruction -Medications: laxatives -Digital examination of rectum: increased resting and squeezing tone of anal sphincter -Caused by dysfunction of? a)Involuntary relaxation of the external anal sphincter b)Involuntary relaxation of the iliococcygeus muscle c)Involuntary relaxation of the pubococcygeus muscles d)Voluntary relaxation of the internal anal sphincter e)Voluntary relaxation of the puborectalis muscle f)Voluntary relaxation of the coccygeus muscle

answer: Voluntary relaxation of the puborectalis muscle -Primary constipation due to inability to coordinate relaxation of pelvic muscles that are involved in defecation -Puborectalis muscle, part of levator ani muscle group= must be voluntarily relaxed to facilitate defecation -This patient= pelvic dyssynergia, which leads to increased sphincter tonus and inability to empty the anal canal -Normally, defecation is enabled by simultaneous voluntary contraction of abdominal muscles and relaxation of the external anal sphincter and puborectalis muscle option A= voluntary option B= part of levator ani, muscle group important for maintaining continence and providing stability of abdominal and pelvic organ; voluntary; no role in defecation process option C= part of levator ani group; dysfunction can lead to increased tonus on digital rectal examination; not directly involved in defecation; voluntary option D= involuntary option F= part of pelvic floor, stabilizes intraabdominal organs, and pulls coccyx forward after defecation; not involved in defecation process

48 years woman -Fever, vomiting, abdominal pain (1 day) -2 weeks (in Guatemala)= emergency appendectomy under general inhalational anesthesia -During surgery= 1 unit of packed RBCs -Fever, high BP, jaundice and tender hepatomegaly -High ALP, AST, bilirubin and direct bilirubin; anti-HAV IgG and Anti-HBs positive -Biopsy= massive centrilobular necrosis -Underlying cause? a)Excessive lysis of red blood cells b)Acalculous inflammation of the gallbladder c)Virus-mediated hepatocellular injury d)Trauma to the bile duct e)Adverse effect of anesthetic f)Gram negative bacteria in the bloodstream

answer: adverse effect of anesthetic -Surgery + jaundice + fever + vomiting + tender hepatomegaly + highly elevated transaminases and centrilobular necrosis= postoperative hepatitis -Highly suggestive of halothane hepatitis -Also leads to eosinophilia and slightly raised ALP -Symptoms began 13 days after surgery= consistent with timeframe for postoperative onset of halothane hepatitis (2 days- 3 weeks) option A= hemolytic transfusion reaction; fever, chills, nausea, flank pain, allergic symptoms would appear shortly after transfusion; labs= indirect hyperbilirubinemia option B= increased ALP and jaundice; unlikely with absence of positive Murphy sign and evidence on US= gallbladder wall thickening, pericholecystic fluid); also, here elevated AST (not consistent with this) option C= hepatitis panel= HAV that resolved; resolved or vaccinated for HBV; also, predominantly affects periportal region option D= common complication of upper abdominal surgery, especially cholecystectomy; leads to cholestatic jaundice; rare after appendectomy; liver biopsy= intraparenchymal cholestasis, manifesting as bile pigment accumulation option F= gram-negative sepsis= complication of surgery that can manifest with acute hepatitis (liver damage secondary to endotoxin release); patient would have hypotension and high spiking fevers

52 years woman -Burns over 45% of her body -Acute distress but fully oriented -Aggressive IV fluid resuscitation and transferred to ICU of burn center -20 hours later, she has large tarry black stools and develops hypotension and tachycardia -She dies; underlying cause of tarry black stools? a)Decreased prostaglandin synthesis b)Erosion of tortuous submucosal arteriole c)Bacterial colonization of the gastric antrum d)Decreased gastric blood flow e)Increased stomach acid production

answer: decreased gastric blood flow -Tarry stools + hypotension + tachycardia prior to death= acute upper GI bleed (precipitated by extensive burn injury) -Curling ulcers are a subtype of stress gastritis seen in patients with extensive burns and occur due to hypovolemia and subsequent hypoperfusion of the stomach -Resultant ischemic tissue injury to stomach epithelium= interruption of normal mucosal barrier of stomach and ulcer formation -PPI should be administered in patients with extensive burns to prevent formation of Curling ulcer option A= NSAIDs option B= Dieulafoy lesion; rare and not associated with burns option C= H. pylori option E= in response to increased vagal stimulation (Acetylcholine); in severe brain injury (Cushing ulcer)

59 years man -Alcohol daily; abdominal swelling and dyspnea -Jaundice, pallor, hepatomegaly, gynecomastia, protuberant abdomen with fluid wave and shifting dullness -Marker to monitor? a)Chromogranin A b)Calcitonin c)Lactate dehydrogenases d)Desmin e)S-100 protein f)Alpha-fetoprotein g)Cancer antigen 125 h)Cancer antigen 15-3 i)Carcinoembryonic antigen j)Cancer antigen 19-9 k)Beta-human chorionic gonadotropin

answer: alpha-fetoprotein -signs= alcoholic liver cirrhosis increased risk of HCC -80% cases of HCC occur due to cirrhosis -Monitor alpha-fetoprotein= diagnosis and recurrence monitoring -AFP= marker for yolk sac tumors (endodermal sinus tumors), mixed germ cell tumors, and ataxia telangiectasia option A= Neuroendocrine origin like carcinoid tumors or small cell lung carcinoma Option B= medullary thyroid carcinoma option C= testicular germ cell tumors, ovarian tumors (dysgerminoma), lymphoma, Ewing sarcoma; assess stage and treatment efficacy but not screening option D= skeletal muscle tumors (e.g., rhabdomyosarcoma) option E= neural crest cells (e.g., melanoma, schwannoma) option G= ovarian cancer option H= along with cancer antigen 27-29)= breast cancer option I= colorectal and pancreatic cancer; monitor progression and treatment efficacy; not for diagnosis option J= pancreatic and gastric cancer; evaluate treatment not diagnosis option K= choriocarcinomas; germ cell tumors; tumor stage, treatment efficacy and prognosis, not diagnosis

75 years man -Fatigue, weight loss, abdominal pain -Previously worked in a factory that produces plastic pipes -RUQ tenderness; biopsy= pleomorphic spindle cells that express PECAM-1 -Diagnosis? a) cavernous hemangioma b) kaposi sarcoma c) angiosarcoma d) hepatocellular carcinoma e) hepatic adenoma f) cholangiocarcinoma

answer: angiosarcoma -association: exposure to vinyl chloride (in plastic), arsenic and thorium dioxide Immunophenotyping: PECAM-1 (CD31)= vascular antigen= angiosarcoma option B= malignant spindle cell tumor; caused by HHV-8, associated with HIV; typically affects skin, can involve GI, oral cavity or respiratory system; patients present with solitary or multiple purplish, nodular, submucosal, painless plaques

38 years man -fever, yellowing of skin, nausea (1 day) -Returned from Brazil and Paraguay (had high fever there which resolved) -Jaundice, epigastric tenderness, petechiae over his trunk -Dark brown emesis and anuria (after admission)= dies -Liver biopsy= eosinophilic degeneration of hepatocytes with condensed nuclear chromatin -Process? a) necrosis b) regeneration c) apoptosis d) proliferation e) steatosis

answer: apoptosis -Patient passed 3 clinical stages of yellow fever (infection, remission, intoxication); endemic in South America; biopsy= Councilman bodies -Councilman bodies= apoptotic hepatocytes appear shrunken and deeply eosinophilic with condensed chromatin (pyknosis) and a fragmented nucleus (karyorrhexis) option A= swollen with prominent membrane blebs and are surrounded by numerous macrophages

34 years man -Foul smelling diarrhea, fatigue, bloating, weight loss (6 months) -Conjunctival pallor and inflammation of corners of mouth; low Hb -Diffuse tenderness to palpation without guarding/rebounding -Intestinal biopsy= villous atrophy, crypt hyperplasia, intraepithelial lymphocytosis; Improve symptoms? a)Treatment with ceftriaxone b)Administration of infliximab c)Avoidance of certain types of cereal grains d)Surgical resection of the colon e)Reduced intake of milk proteins f)Supplementation of pancreatic enzymes

answer: avoidance of certain types of cereal grains biopsy= celiac disease -All signs point to Celiac (T1D= risk factor); can confirm with positive celiac serology -Bimodal distribution= infants and adults in 3rd-4th decade of life -Treatment: lifelong gluten free diet (wheat, rye, barely, spelt) option A= Whipple disease; PAS-positive macrophages; associated with cardiac symptoms (e.g., valve insufficiencies), arthralgias and neurological signs (e.g., myoclonia, ataxia, oculomotor impairment option B= Crohn disease option D= ulcerative colitis option E= lactose intolerance option F= pancreatic exocrine insufficiency; intestine biopsy= normal

56 years woman -Intermittent upper abdominal pain few hours after meals (relived by food intake) -Endoscopy: ulcer on posterior wall of duodenal bulb -Biopsy from edge of ulcer: hyperplasia of submucosal glandular structures -Hyperplasia of cells causes? a) glycoprotein synthesis b) antigen presentation c) lysozyme secretion d) cholecystokinin secretion e) nutrient absorption f) hydrochloric acid secretion g) bicarbonate secretion

answer: bicarbonate secretion -Brunner glands are in submucosa of duodenum and secrete alkaline, bicarbonate rich fluid to neutralize stomach acid -Duct openings deliver secretions into base of crypts of Lieberkühn -PUD can lead to compensatory hyperplasia of Brunner glands option A= goblet cells, located in mucosa, in jejunum and ileum more option B= presented by Peyer's patches in lamina propria option C= Paneth cells; in base of crypts of Lieberkühn in mucosa of small intestine option D= I cells; duodenal and jejunal mucosa; stimulates gallbladder contraction, sphincter of Oddi relaxation; and secretion of enzymes and bicarbonate by pancreas; slows gastric emptying option F= parietal cells in gastric mucosa

49 years woman Abdominal pain (2 months), nausea, vomiting, early satiety RUQ tenderness; liver span (16cm) Ultrasonography: hyperechoic mass in left lobe of liver Biopsy= blood-filled vascular spaces (variable size) lined by single epithelial layer Diagnosis? a) hepatocellular adenoma b) alveolar echinococcosis c) polycystic kidney disease d) cavernous hemangioma e) focal nodular hyperplasia f) angiosarcoma g) hepatocellular carcinoma

answer: cavernous hemangioma -Biopsy= benign vascular tumor -Most common benign hepatic tumor (women 30-50 years usually) -Asymptomatic; if large enough= compress adjacent bowel (early satiety) and stretch the Glisson capsule (upper abdominal pain) -Highly vascular= hyperechoic masses on US -Biopsy should be avoided due to risk of hemorrhage option A= long-term use of oral contraceptives in women; usually asymptomatic, detected incidentally; US= well-demarcated heteroechoic intraparenchymal lesion; histology: benign hepatocytes that contain glycogen and lipid deposits option B= affects liver; abdominal pain and hepatomegaly; imaging: anechoic cysts with or without septae and areas of calcification; histology= walls of cysts are seen as lamellated membranes, surrounded by chronically inflamed hepatic parenchyma, and areas of calcification option C= multiple hepatic cysts; RUQ pain, hepatomegaly; imaging= multiple anechoic unilocular cysts; histology= cysts lined with cuboidal biliary epithelium and surrounded by normal hepatic parenchyma option E= benign tumor; large= hepatomegaly, abdominal pain, early satiety; imaging: intrahepatic mass of variable echogenicity often with a central scar; biopsy= benign hepatocytes arranged in nodules; nodules separated by fibrous septa and a central scar option F= men > 60 years; exposure to arsenic, vinyl chloride or thorium dioxide; RUQ pain, early satiety, hepatomegaly; also causes weight loss, fatigue, ascites; US= multiple, large, intraparenchymal lesions with necrotic areas; histology= vascular spaces lined with dysplastic endothelium that infiltrates hepatic cords and sinusoids option G= upper abdominal pain, early satiety, hepatomegaly; RISK FACTOR= CHRONIC LIVER DISEASE; US= areas of necrosis and calcification; histology= pleomorphic hepatocytes with eosinophilic cytoplasm; prominent nucleoli and mitotic figures; areas of necrosis and neovascularization

63 years man -Diverticular disease; painless rectal bleeding, dizziness -high pulse; low BP; pale -Colonoscopy: profuse diverticular bleeding= endoscopic hemostasis -Fluid resuscitation= hemodynamically stable -Next day- labs: low hemoglobin, high WBCs, high urea nitrogen (60mg/dl), high creatinine (2.1mg/dl), mildly high direct bilirubin, extremely high ALT and AST -Cells of liver damaged first? a) hepatic stellate (Ito) cells b) periportal hepatocytes c) hepatic sinusoidal endothelial cells d) hepatic kupffer cells e) midzonal hepatocytes f) centrilobular hepatocytes

answer: centrilobular hepatocytes -Extremely high AST and ALT + normal ALP + mildly high direct bilirubin + previous hemodynamic instability + evidence of prerenal acute kidney injury (BUN: Cr >20:1)= ischemic hepatitis -Liver= hepatic lobules (anatomic unit), portal lobules (bile-synthetic functional unit) and hepatic acini (microvascular unit) -Hepatic acinus (diamond shaped zone) between 2 neighboring central veins (long axis) and two opposing portal triads (short axis) -Within the acinus, blood enters through the portal triad (75% deoxygenated blood from hepatic portal vein; 25% oxygenated from hepatic artery) and drains through sinusoidal capillaries into central vein -Hepatic acini can be divided into 3 zones, which are distinguished based on degree of oxygenation -Zone 3 contains centrilobular hepatocytes and surrounds central vein --> because zone 3 receives oxygen-poor blood, most sensitive to ischemia option A= located within space of Disse; secrete extracellular matrix to protect and facilitate healing of liver in response to transient hepatic injury (ischemic hepatitis); low metabolic rate so resistant to ischemia than hepatocytes option B= Zone 1; most susceptible to viral hepatitis and toxic substance ingestion; receives oxygen rich blood and is least sensitivity to ischemic injury option C= drain blood from portal triad into central vein; extremely sensitive to reperfusion injury, which may lead to their destruction, induce portal hypertension, and worsen hepatic fibrosis; lower metabolic rate so more resistant to ischemic injury than hepatocytes option D= located within sinusoids; clear aged erythrocytes, infectious particles, debris; lower metabolic rate, more resistant to ischemia than hepatocytes option E= Zone 2; most susceptible to hepatocellular necroptosis (with councilman bodies) due to yellow fever; receives oxygen intermediate blood= neither first or last damaged by ischemia

45 years man -Fever, malaise, RUQ pain (tenderness) -High WBCs (90% neutrophils), high ALP -US= 3-cm hypoechoic lesion in the right lobe of the liver with hyperemic rim -Cause? a) colorectal cancer b) infectious endocarditis c) echinococcosis d) diverticulitis e) cholangitis f) perinephric abscess g) candidiasis

answer: cholangitis -Fever, malaise, RUQ pina + neutrophilic leukocytosis + US= liver abscess -Ascending infection from biliary tract pathology (e.g., cholangitis) is the most common cause of hepatic abscesses -Caused by bacteria (polymicrobial) with E. coli being most frequent -May also result from intraabdominal infections (e.g., acute appendicitis, peritonitis) or hematogenous spread from systemic circulation (e.g., sepsis) option B= no cardiac manifestations (e.g., new or changed heart murmur, heart failure signs), and signs such as Janeway lesions, Osler nodes, and Roth spots; moreover, IE is unlikely without risk factors like predamaged or prosthetic heart valve, IV drug abuse or recent dental surgery option C= hepatic cyst; tapeworm exposure (e.g., contact with dogs or other potentially infected definitive hosts); labs (eosinophils high, leukopenia) and US (anechoic, well-defined cysts with eggshell calcifications

67 years man RLQ pain CT: mass in terminal ileum biopsy: numerous small cells of similar shape and size (monomorphic cells) arranged in a rosette pattern cells stain with? a) desmin b) chromogranin A c) vimentin d) p-glycoprotein e) cytokeratin f) glial fibrillary acid protein

answer: chromogranin A - biopsy= carcinoid tumor -Chromogranin A= protein found in neuroendocrine cells in pancreas (islet cells), GI, lungs, and adrenal glands -Chromogranin A, synaptophysin, and neuron specific enolase can be used option A= protein in muscle (cardiac, skeletal, smooth); usually in muscle sarcomas (rhabdomyosarcoma option C= a)protein found in cells of mesenchymal origin, including cells of musculoskeletal system (osteoblasts, chondroblasts, fibroblasts, adipocytes, endothelial cells); mesenchymal tumors) option D= in most cells of body; several tumors (HCC, colon cancer, adrenocortical tumor option E= protein of epithelial origin (lining cells of GI, respiratory, urinary and reproductive tracts); epithelial tumors (SCC, adenocarcinoma option F= CNS; astrocytes, schwann cells and oligodendrocytes; GRAP would be positive in astrocytoma, glioblastoma, etc.)

49 years woman black stones cause? a) diabetes mellitus b) primary hyperparathyroidism c) chronic hemolytic anemia a d) menopausal symptoms e) morbid obesity

answer: chronic hemolytic anemia -Increased RBC breakdown= increased indirect bilirubin= precipitates in gallbladder as calcium bilirubinate (black stones) -Cause: chronic hemolytic anemia

57 years man -High AST, ALT, GGTP; HBsAg (positive) -Liver biopsy: lymphocytic infiltrates, piecemeal necrosi, ground glass hepatocytes -Characteristic of? a) acute inflammation b) ischemic necrosis c) malignant transformation d) granulomatous inflammation e) chronic inflammation

answer: chronic inflammation due to biopsy findings option A= lacks Councilman bodies, stellate cells and diffuse apoptosis option B= due to acute hepatitis; appear as necrotic hepatocytes around central venules option C= no nuclear atypia, dedifferentiation option D= in TB and schistosomiasis

34 years woman -Crohn disease; nausea, bloating, epigastric pain (after meals) radiates to shoulder -4 months ago= ileocecal resection (intestinal obstruction) -US= multiple echogenic foci with acoustic shadows in gallbladder -Cause? a) increased bilirubin production b) increased hepatic cholesterol secretion c) increased activity of beta-glucuronidase d) decreased fat absorption e) decreased motility of the gallbladder f) decreased biliary concentration of bile acids

answer: decreased biliary concentration of bile acids -Crohn disease= Decreased concentration of bile acids (ileitis and/or terminal ileum resection) -Bile acids normally absorbed in terminal ileum (enterohepatic circulation) -Decreased bile acid/ cholesterol ratio= supersaturation of cholesterol (stones) option A= hemolysis= black stones option B= increased estrogen circulation (pregnancy)= yellow stones option C= beta-glucuronidase production by bacteria in biliary tract; deconjugates bilirubin in gut for reabsorption= excess bilirubin with calcium= brown pigment stones option D= fatty acids bind calcium; calcium not available to bind oxalate to aid in excretion= hyperoxaluria= calcium oxalate kidney stones option E= stone formation; but symptoms here makes it unlikely

53 years man liver transplantation (1 year ago due to alpha-amanitin poisoning) biopsy= decreased hepatic duct density histology? a) graft vessel vasculitis b) neoplastic cells containing bile c) fibrinoid necrosis d) interstitial fibrosis e) viral inclusions

answer: interstitial fibrosis -One year after transplant= chronic graft rejection -Occurs at both cellular and humoral level in response to donor peptides -Interstitial fibrosis, atrophy of parenchyma, vascular smooth muscle proliferation and ductopenia option A= acute rejection within 6 months of transplant due to antibody formation against donor MHCs; histology: vasculitic changes of graft vessels and interstitial lymphocytic infiltrate option C= hyperacute rejection within 48 hours; reaction of pre-existing antibodies against donor antigens= complement activation and widespread thrombosis and necrosis; histology: fibrinoid necrosis, vessel thrombosis, tissue ischemia

49 years man -Jaundice and scleral icterus -Urinalysis= elevated bilirubin and low urobilinogen -Cause? a) Absent UDP-glucuronosyltransferase activity b) Increased hemoglobin breakdown c) malignant growth in the pancreatic body d) increased intestinal bilirubin reabsorption e) defective heaptic bile excretion f) presence of stones within the gallbladder

answer: defective hepatic bile excretion -Unconjugated bilirubin conjugated in liver with glucuronic acid -Secreted via bile and converted to urobilinogen by intestinal bacteria -Defective hepatic bile excretion (e.g., due to bile duct stenosis or choledocholithiasis)= conjugation continues (unconjugated bilirubin remains normal) but conjugated bilirubin can't be secreted -Conjugated bilirubin accumulates in blood and is excreted by kidney option A= Gilbert and Crigler Najjar; unconjugated high option B= unconjugated high and increased conjugation would lead to high conversion of conjugated bilirubin to urobilinogen option D= conjugated hyperbilirubinemia; would show both increased bilirubin and urobilinogen excretion

48 years woman -Fatigue, generalized pruritus, jaundice -Hypothyroidism -High direct bilirubin; high ALP -Antimitochondrial antibodies and anti-thyroid peroxidase antibodies -Cause? a) hepatocellular accumulation of copper b) idiopathic hepatocellular accumulation of fat c) neoplasia of ampulla of Vater d) destruction of intrahepatic bile ducts e) autoimmune-mediated destruction of hepatocytes f) inflammation and fibrosis of biliary tree

answer: destruction of intrahepatic bile ducts jaundice/pruritus + high ALP, high direct bilirubin + antimitochondrial antibodies + woman= PBC (associated with hashimoto's and other autoimmune diseases) -Histology: destruction of intrahepatic bile ducts option A= wilson disease option B= non-alcoholic fatty liver option E= autoimmune hepatitis option F= PSC; men; pANCA antibodies

67 years man -Fatigue, weight loss (4 months), jaundice -Hepatomegaly; elevated alpha-fetoprotein; prolonged prothrombin time -Genetic analysis: G:C to T:A transversion in codon 249 of gene coding for TP53 protein -Most specific risk factor? a) alcoholism b) hepatitis C infection c) dietary aflatoxin exposure d) schistosomiasis e) anabolic steroid use f) hemochromatosis

answer: dietary aflatoxin exposure -Signs of HCC and G:C transversion= history of aflatoxin B1 ingestion -Aflatoxin is produced by some strains of the mold Aspergillus -A metabolite of aflatoxin B1 interferes with proper DNA transcription of TP53 gene through an inactivating mutation, resulting in the base transversions here -Mutation found in up to 60% of individuals with HCC in areas with high levels of exposure to aflatoxin B1 option A= liver cirrhosis; associated with 80% of cases of HCC; no specific mutation of TP53 option B= due to chronic HCV, via viral and non-viral pathophysiological steps; no G:C to A:T transversion option D= potentiates dysplastic changes induced by viral hepatitis or carcinogenic substance like diethylnitrosamine; not associated with transversion option E= associated with adenoma; may undergo malignant transformation option F= associated with mutation in HFE gene on chromosome 6; most common mutations are substitution of cysteine to tyrosine due to G to A transition at codon 282 or substitution of histidine with aspartate due to C to G transversion at codon 63

age: 67 years woman -Episodic abdominal pain, nausea, vomiting -Conditions: CAD, T2D; high BMI -Fever, dry mucous membranes, abdominal distention, hyperactive bowel sounds -US= air in biliary tract -Obstruction where? a) third part of the duodenum b) distal ileum c) hepatic duct d) proximal jejunum e) pancreatic duct

answer: distal ileum -Risk factors of gallstone (e.g., obesity, age> 40, female gender) -Pneumobilia (US)= gallstone ileus -Rare complication of cholecystitis= Cholecystoenteric fistula forms (air can enter biliary tree) -Gallstones can pass through fistula to ileocecal valve (narrowest part) -Other causes of distal ileal obstruction= fecaliths, Meckel diverticulum, intussusception option A= SMA syndrome; narrowing of aortomesenteric angle option C= can be obstructed by gallstone= Mirizzi syndrome; abdominal pain, fever, nausea, vomiting; manifests with jaundice too and no obstruction option D= due to bowel adhesions (prior abdominal surgery), or intussusception option E= due to gallstone; no pneumobilia or small bowel obstruction

newborn -Bilious vomiting; lethargic, fontanelles are sunken -X-ray: double bubble sign -Congenital obstruction of? a) esophagus b) common bile duct c) ileum d) pylorus e) duodenum

answer: duodenum Double bubble sign (air in distended stomach and proximal duodenum and no gas in distal bowel)= congenital duodenal atresia -Causes bowel obstruction distal to major duodenal ampulla -Leads to= abdominal distention, bilious vomiting and features of dehydration (e.g., sunken anterior fontanelle) -25% associated with Down syndrome option A= tracheoesophageal fistula; non-bilious vomiting; gastric distention from air passing from the trachea into distal esophagus segment option B= biliary atresia; cholestatic jaundice option C= ileal atresia; abdominal distention and vomiting in first few days of life; X-ray= dilated loops of small bowel extending to level of obstruction option D= hypertrophic pyloric stenosis; non-bilious vomiting; X-ray= stomach distended with air, but no air in duodenum; becomes symptomatic, 3-5 weeks after life

61 years man -Fatigue, weight loss (6 months) -IV drugs (in his 20s), has HCV; smoker (35 years) -Father: died (colon cancer) -Scleral icterus; several telangiectasias; liver (firm and nodular) -Labs: low hemoglobin, high WBCs -US= isoechoic lesion with hypoechoic border in liver - most likely finding? a)Bacteremia b)Bile duct strictures alternating with dilation c)Elevated antimitochondrial antibodies d)Elevated alpha-fetoprotein e)Lesion with eccentric calcification on chest CT f)Elevated carcinoembryonic antigen g)Annular colonic lesion on colonoscopy

answer: elevated alpha-fetoprotein -Liver cirrhosis (firm and nodular liver, scleral icterus and telangiectasia) and hepatitis infections (HCV) (risk factors for HCC) -US= HCC -Further symptoms (advanced disease)= weight loss, cachexia, ascites and jaundice option A= pyogenic liver abscesses; most commonly develop secondary to biliary tract obstructions but also by bacteremia (IV drug users, he is not anymore; no fever, no malaise, no RUQ pain, no leukocytes (unlikely); fluid-filled hyperechoic lesion option B= PSC option C= PBC option E= lung cancer; he has no symptoms of lung cancer like cough, hemoptysis and dyspnea; can metastasize to liver but usually multiple lesions option F= colorectal and pancreatic cancer; both can metastasize to liver but here solitary lesion option G= colorectal cancer; can spread to liver via portal vein; he lacks typical symptoms (hematochezia, changes in bowel habits, iron deficiency anemia)

36 years woman -Difficulty swallowing, burning sensation in chest (2 years) -PPIs and antacids (2 months) without relief -She uses low dose inhaled corticosteroids for allergic asthma -High BMI; upper endoscopy shown -Diagnosis? a) gastroesophageal reflux disease b) herpes esophagitis c) candida esophagitis d) drug-induced esophagitis e) eosinophilic esophagitis f) cytomegalovirus esophagitis

answer: eosinophilic esophagitis -endoscopy= white, circumferential mucosal lesions (trachealization of the esophagus), linear, longitudinal furrows -+ history of atopy + resistance to PPIs= eosinophilic esophagitis -May also present with food impaction, reflux and vomiting -Biopsy= eosinophilic infiltrate option A= usually affects distal esophagus and improves with PPIs; endoscopy= normal, if chronic then esophageal ulcers, strictures, Barrett esophagus or esophageal adenocarcinoma option B= refractory to PPIs; usually in immunocompromised; endoscopy= show vesicles and multiple small superficial round/ovoid punched out ulcers in mid esophagus that may coalesce to form diffuse superficial ulcers option C= refractory to PPIs; in immunocompromised; endoscopy= white mucosal plaque-like lesions option D= refractory to PPIs; endoscopy= punched out ulcer surrounded by normal mucosa that may be mildly inflamed; NSAIDs, bisphosphonates, tetracyclines, iron= not taken by patient option F= refractory to PPIs; in immunocompromised; endoscopy= large linear ulcerations in distal esophagus that are shallow

30 years man -Shortness of breath, fever, cough (5 days); epigastric pain -Smoker, cocaine on weekends -ICU= management of pneumococcal sepsis -Hematemesis (one week later) -Endoscopy: multiple shallow hemorrhagic lesions (fundus and greater curvature) -Biopsy: patchy epithelial defects that do not extend beyond muscularis mucosa -Cause? a) type B gastritis b) dieulafoy lesion c) cushing ulcer d) penetrating ulcer e) curling ulcer f) erosive gastropathy

answer: erosive gastropathy -Epigastric pain + hematemesis after pneumococcal sepsis + defects not beyond muscularis propria= erosive gastropathy -Ulcers= extend beyond muscularis mucosa (deeper) -Acute erosive gastropathy= ischemia of mucosa due to hypovolemia (e.g., sepsis) and exposure to injurious substances (smoking, cocaine use) -Hemorrhage and loss of superficial epithelium= compromising normal protective barrier= acid penetrates through the mucosa= damages vasculature= exacerbates mucosal ischemia= release of inflammatory mediators= erosions option A= H. pylori option B= erosion of proximal stomach related to an aberrant submucosal artery; hematemesis and epigastric pain; biopsy: intact submucosa; usually SINGULAR and along lesser curvature near esophagogastric junction; endoscopy= dilated submucosal vessel option C= stress ulcer; brain injury; solitary and tendency to perforate; extends beyond muscularis mucosa option D= extends into serosa; peritoneal signs option E= physiological stress (severe burns); tendency to perforate; extends beyond muscularis mucosa

56 years man -Intermittent retrosternal chest pain -Endoscopy: salmon pink mucosa extending 5cm proximal to the gastroesophageal junction -Biopsy: nonciliated columnar epithelium with numerous goblet cells -Cause? a)Neoplastic proliferation of esophageal epithelium b)Esophageal exposure to gastric acid c)Atopic inflammation of the esophagus d)Hypermotile esophageal contractions e)Fungal infection of lower esophagus f)Incomplete relaxation of lower esophageal sphincter

answer: esophageal exposure to gastric acid - Barrett esophagus= most common cause= gastric acid in esophagus option A= increased risk of adenocarcinoma; doesn't have it yet option C= eosinophilic esophagitis option D= nutcracker esophagus, diffuse esophageal spasm option E= candida esophagitis option F= achalasia, absent or decreased myenteric plexus

64 years woman -Nausea, vomiting, retrosternal pain that radiates to back (2 hours) -Tenderness in epigastric area; CT: extraluminal gas in mediastinum (pneumomediastinum) -Diagnosis? a) esophageal rupture b) pulmonary embolism c) aortic dissection d) acute myocardial infarction f) Mallory-Weiss syndrome g) aspiration pneumonia

answer: esophageal rupture -Boerhaave syndrome= caused by excessive vomiting and retching (e.g., result of high alcohol intake) -Increased intrathoracic pressure= elevated intraesophageal pressure= full-thickness esophageal wall rupture -History of vomiting, severe retrosternal pain (radiates to back) -Auscultation= crackling or crunching sound (Hamman sign) due to presence of air in soft tissue spaces option B= sudden-onset, severe chest pain; dyspnea, cough, tachycardia, hypotension; CT imaging (helical spiral CT/CT pulmonary angiography) classically shows an intraluminal filling defect of pulmonary arteries; the presence of wedge-shaped infarction with pleural effusion is highly specific finding option C= retrosternal pain that radiates to back; history of hypertension and usually present with severely elevated BP (or hypotension in cases of shock and/or cardiac tamponade); asymmetrical BP readings between arms and syncope; CT= intimal flap that separates a true and false lumen option D= nausea, vomiting, retrosternal pain; CT angiography not commonly used= atherosclerotic plaques and coronary artery stenosis option E= sudden-onset, severe chest pain; dyspnea, reduced/absent breath sounds, hyperresonance on percussion, decreased fremitus on ipsilateral side; CT= air pockets in pleural space option F= history of forceful vomiting (alcohol use); retrosternal pain and epigastric tenderness; hematemesis; EGD is preferred not CT= single, longitudinal tear in the mucosa at esophagogastric junction option G= chest pain; dyspnea, cough, fever, crackles on auscultation and hypoxemia; radiology= infiltrates in dependent parts of the lung

4 years girl -Abdominal pain 1 hour after drinking bottle of rust remover -Vomited once (nonbloody); odynophagia; mild erythema of epiglottis and heavy salivation -Most likely long-term complication? a) esophageal webs b) esophageal strictures c) barrett esophagus d) thyroglossal fistula e) mallory-weiss tears f) oral cavity cancer

answer: esophageal strictures -Caustic liquid ingestion (highly alkaline (liquefactive necrosis or acidic (coagulative necrosis)) -long term complications à esophageal strictures (most common), esophageal ulcers, esophageal cancer option A= thin membranes of normal esophageal tissue that protrude into esophagus; dysphagia, odynophagia and food impaction; congenital (Plummer-Vinson syndrome) or acquired (e.g., nutritional deficiencies option D= congenital remnant of thyroid diverticulum; mediastinal and tracheoesophageal fistulas may develop in patients with severe caustic liquid injury; especially in those with perforation option F= associated with smoking, alcohol consumption, poor oral hygiene; and HPV (16, 18, 31, and 33)

50 years man -Intermittent watery diarrhea, abdominal pain, weight loss (8 months) -Episodic pain of ankle, wrist and knee joints (5 years) -Histology= foamy macrophages in lamina propria with PAS-positive inclusions -Further evaluation? a)Multinucleated trophozoites b)Anti-tissue transglutaminase antibodies c)Anti-cyclic citrullinated peptide antibody d)Anti-saccharomyces cerevisiae antibodies e)Intracellular, gram-positive bacilli f)Low serum TSH and high free T4 concentrations

answer: intracellular, gram-positive bacilli -Chronic polyarthritis preceding diarrhea, abdominal pain and weight loss + histology= Whipple disease -Organism: Tropheryma whippelii -Lethal if left untreated, IV antibiotics (ceftriaxone or penicillin plus streptomycin) for 14 days followed by maintenance therapy with trimethoprim-sulfamethoxazole (TMP-SMX) for 1 year option A= Entamoeba histolytica or Giardia lamblia option B= Celiac option C= rheumatoid arthritis option D= Crohn option F= hyperthyroidism

57 years man -Worsening epigastric pain, weight loss (2 months) -Painful, red blotches on medial aspect of right calf -Smoker (35 years); erythematous nodules on right ankle and left antecubital fossa -Endoscopy: mass in antrum of stomach -Biopsy= disorganized, mucin-secreting cells with surrounding fibrosis -Cells originate from? a)Lymphoid tissue in the terminal ileum b)Mature hepatocytes in the liver c)Exocrine ducts in the body of the pancreas d)Squamous epithelium in the proximal esophagus e)Glandular epithelium in the sigmoid colon

answer: exocrine ducts in the body of the pancreas -Epigastric pain + smoker + erythematous nodules + biopsy= pancreatic adenocarcinoma (CA19-9 is the marker) -Pancreatic adenocarcinoma (glandular mucin secreting cells typical; derived from exocrine cells of pancreas) -Pancreatic head is most common site (usually manifest with painless jaundice and pruritus secondary to obstruction of CBD)= lacks these symptoms so cancer probably originated from body or tail -Pancreatic adenocarcinoma does not manifest until it has metastasized option A= MALT lymphoma; dense collections of lymphoid cells; associated with chronic autoimmune diseases option B= HCC option D= eSCC option E= colorectal cancer; age and smoking support this diagnosis; unlikely because manifests with iron deficiency anemia, hematochezia, and changes in bowel habits

22 days boy -Poor feeding, lethargy, abdominal protrusion -25th percentile height, 50th percentile weight, 95th percentile head circumference -Scleral icterus, pale facies, enlarged tongue -Protruding mass at abdominal midline; skin covering protrusion (normal) -When newborn cries= mass enlarges but is easily reducible -Cause? a) failed convergence of the lateral embryogenic folds b) failed obliteration of the vitelline duct c) congenital weakness of the linea alba d) persistence of a patent processus vaginalis e) failed spontaneous closure of the umbilical ring

answer: failed spontaneous closure of the umbilical ring -Congenital umbilical hernia -Midgut develops outside abdominal cavity until 2nd trimester, when it physiologically herniates back into the abdomen -if umbilical ring fails to close or fascia in this region is underdeveloped= abdominal content may bulge through the umbilicus -Most common in children with chromosomal abnormalities (e.g., Down and Edwards syndrome) or congenital hypothyroidism (here) -Most resolve spontaneously by 5 years of age option A= a)leads to gastroschisis (paraumbilical herniation of intestines, not covered by sac/skin) or omphalocele (organs herniate via midline abdominal wall into hernial sac covered by peritoneum only) option B= Meckel's diverticulum option C= epigastric hernia, occurs in abdominal midline (between xiphoid process and umbilicus); hernias typically manifest during adulthood, not during neonatal period option D= indirect inguinal hernia; but it would manifest in the groin not abdomen midline

38 years man -Several ulcers in the duodenum and upper jejunum as well as thick gastric folds -Gastric pH<2; biopsy of ulcer= no organisms -Test to confirm diagnosis? a) 24-hour esophageal pH monitoring b) fasting serum gastrin level c) urine metanephrine levels d) urea breath test e) serum vasoactive intestinal polypeptide level

answer: fasting serum gastrin level -Zollinger-Ellison syndrome -Baseline gastrin level >10-fold increase= gastrinoma (especially with pH<2) -Secretin stimulation test may be used (done without acid suppression therapy)= should increase gastrin option A= GERD option C= catecholamine metabolites; pheochromocytoma; ZES associated with MEN-1 while pheochromocytoma with MEN-2A and 2B option D= H. pylori option E= VIPoma; associated with MEN-1

65 years man -Fever and disorientation (1 day) -Alcoholic; high pulse and low BP; jaundice, palmar erythema, spider nevi on chest; dilated veins on anterior abdominal wall; and 2+ edema of lower extremities -Shifting dullness to percussion; low albumin; high bilirubin, high PT time -Hepatitis serology= negative; CT: nodular liver - histology? a)Accumulation of iron in hepatocytes b)Hepatocyte swelling and necrosis with neutrophilic infiltration c)Ground-glass hepatocytes with cytotoxic T cells d)Fibrous bands surrounding regenerating hepatocytes e)Macrovesicular lipid accumulation in hepatocytes f)Hepatocyte swelling with Councilman bodies and monocyte infiltration g)Misfolded protein aggregates in hepatocellular endoplasmic reticulum

answer: fibrous bands surrounding regenerating hepatocytes -All signs of cirrhosis= confirmed by CT -Hepatocyte destruction + hepatic stellate cell activation via inflammatory cytokines= excess collagen fiber production in periportal and pericentral zones -Continuous formation of fibrous bands surrounding regenerating hepatocytes (i.e., fibrosis and nodular regeneration) destroy normal liver architecture= loss of liver metabolic function -Fibrosis= increased intrahepatic vascular tone= portal hypertension= symptoms seen option A= hemochromatosis= cirrhosis; no features of hemochromatosis= diabetes, arthralgia, bronze skin pigmentation option B= alcoholic hepatitis= eventually lead to cirrhosis; but in this stage the liver would be smoothly enlarged, not shrunken or nodular option C= HBV option E= hallmark of hepatic steatosis; can lead to cirrhosis but in this stage, liver is smoothly enlarged option F= acute viral hepatitis option G= alpha-1 antitrypsin deficiency= can lead to cirrhosis; but would also have panacinar emphysema; not caused by alcohol

65 years man -age: 65 years man -Abdominal pain and bloody, mucoid diarrhea (3 days) -Constipation (past 6 months) -History: type 2 diabetes; smoker (35 years) -Fever, high/normal pulse, high/normal BP -Palpation of LLQ= tenderness -CT scan (no mass in LLQ, sigmoid colon thickened and there is gas pouch adjacent); CT shown in question cause? a) focal weakness of the colonic muscular layer b) infiltrative growth in the descending colon c) transmural inflammation of terminal ileum d) twisting of simgoid colon around its mesentery e) decreased perfusion to mesenteric blood vessel f) mucosal ulcerations of the rectum

answer: focal weakness of the colonic muscular layer description of picture:The wall of the sigmoid colon (green hatched overlay) is thickened and there is gas within the adjacent diverticula (red hatched overlay). -Focal weakness of colonic muscular layer= typically at points where vasa recta traverse the intestinal wall= allows for formation of protrusions of mucosal and submucosal layers in response to increased intraluminal pressure (chronic constipation) -Pseudodiverticula= not all layers of intestinal wall are affected -Diverticulosis= asymptomatic (air-filled pouches on CT) -Diverticulitis= acute inflammation of diverticula, manifest with bowel wall thickening on CT, fever, LLQ pain, bloody mucoid diarrhea -risk factors: diet (low in fiber, high in red meat and fat), high BMI, lack of physical activity and smoking (>40 pack years) option B= colon cancer; no mass; this is acute process=3 days; colon cancer slowly progresses option C= Crohn disease; transmural inflammation of terminal ileum, bloody diarrhea, low grade fever, abdominal pain; but also, malabsorption and arthritis erythema nodosum; CT= bowel wall thickening, fat stranding and strictures or fistulas option D= a)sigmoid volvulus; can lead to bowel obstruction, infarction and/or perforation; manifests: episodic abdominal pain, asymmetric abdominal distention, progressive signs of bowel obstruction; severe cases: rectal bleeding, hematemesis, peritonitis; CT= gas-filled loops of bowel, twisting of the mesentery and mesenteric vessels (whirl sign) and gradual narrowing or tapering of sigmoid colon up to levels of obstruction (bird beak sign) option E= acute mesenteric ischemia; caused by arterial embolism (most common), thrombus or low CO; severe acute periumbilical pain; then bloody diarrhea and rectal bleeding; CT: intestinal pneumatosis, mesenteric stranding and bowel wall thickening and distention option F= ulcerative colitis; low-grade fever, abdominal pain, bloody diarrhea; CT= bowel wall thickening, but adjacent air-filled pouches not seen; also UC= ankylosing spondylitis, migratory monoarticular arthritis, erythema nodosum

24 years man -Loose, nonbloody stools (6 weeks), abdominal pain -Tenderness in RLQ; leukocytosis, high ESR -CT= transmural thickening and surrounding fat stranding of discrete regions in terminal ileum and transverse colon extra findings? a)Neutrophilic inflammation of the crypts b)Neutrophil-rich pseudomembranes c)Poorly differentiated gland-forming cells with desmoplasia d)Inflammation of the terminal ileum e)Formation of noncaseating granulomas f)Inflammation limited to mucosa and submucosa g)Presence of pseudopolyps

answer: formation of noncaseating granulomas - Transmural inflammation= Crohn disease= noncaseating granulomas (hallmark) option A= can be in Crohn; more likely in UC option B= pseudomembranous enterocolitis option C= colon cancer option D= can occur in CD but nonspecific option F= UC option G= maybe in CD; more common in UC

34 years woman -Abdominal pain, nausea, vomiting -Abdominal guarding and rigidity; bowel sounds reduced -MRCP= dorsal pancreatic duct draining into the minor papilla and a separate smaller duct draining into major papilla -Spleen is located anterior to left kidney -Disruption of? a)Fusion of the pancreatic buds b)Fusion of visceral and parietal peritoneum c)Rotation of the midgut d)Proliferation of mesenchyme in the dorsal mesentery f)Differentiation of the proximal hepatic diverticulum g)Rotation of the ventral splenic bud

answer: fusion of the pancreatic buds -Fusion of pancreatic ducts= 8th week of development -Distal portion of dorsal pancreatic duct fuses with ventral pancreatic duct= drains into major duodenal papilla via Ampulla of Vater -Proximal portion of dorsal pancreatic duct would degenerate or persist as an accessory pancreatic duct that drain into minor duodenal papilla -If fusion fails= dorsal pancreatic duct would drain into minor duodenal papilla -Most individuals with pancreatic divisum are asymptomatic, but small percentage will present with chronic abdominal pain or pancreatitis option B= pancreas would remain intraperitoneal option C= malrotation; formation of fibrous bands (Ladd bands) may lead to volvulus or acute intestinal obstruction; imaging: small bowel on right side and large bowel on left option D= process of spleen development; spleens position in this patient is normal option E= biliary system development option F= splenic bud located in dorsal mesentery; a ventral splenic bud does not exist; failure of fusion of dorsal primordial splenic buds= supernumerary (accessory) spleen formation; in 10% of population

56 years woman -US= hyperechogenic rim-like calcification of gallbladder wall -Risk for? a)Hepatocellular carcinoma b)Pancreatic adenocarcinoma c)Gallbladder empyema d)Pyogenic liver abscess e)Emphysematous cholecystitis f)Gallbladder carcinoma g)Gallstone ileus h)Acute pancreatitis

answer: gallbladder carcinoma -Porcelain gallbladder -Caused by chronic cholecystitis -Increases risk of gallbladder adenocarcinoma Cholecystectomy is recommended option A= risks: cirrhosis, aflatoxin, hemochromatosis option B= risks: smoking, chronic pancreatitis, diabetes, age> 50, Ashkenazi Jewish, African American option C= collection of pus in gallbladder; risks: cholelithiasis and cholecystitis option D= often develop from biliary obstruction as a result of chronic cholelithiasis option E= risks: cholecystitis; air accumulates in wall due to constriction of blood vessels and subsequent infection with gas forming bacteria option G= biliary-enteric fistula; stone gets impacted to ileum; risks: acute cholecystitis and cholelithiasis option H= risks: alcohol, cholelithiasis

52 years woman -Epigastric abdominal pain (after meal); worsening (6 hours) similar episodes (never lasted this long) -Fever, sudden inspiratory arrest during RUQ palpation -Total bilirubin, ALP, amylase, and AST within normal ranges -Following findings? a)Dilated common bile duct with intrahepatic biliary dilation b)Gas in the gallbladder wall c)Gallstone in the cystic duct d)Fistula formation between the gallbladder and bowel e)Decreased echogenicity of the liver f)Enlargement of the pancreas with peripancreatic fluid

answer: gallstone in the cystic duct -Middle aged woman + postprandial epigastric pain + fever + positive murphy sign= acute cholecystitis -Most cases= obstructing gallstone in cystic duct -US of RUQ is indicated= gallbladder wall thickening, gallbladder wall edema, obstructing gallstone and/or positive murphy sign on US option A= Choledocholithiasis; also, jaundice, elevated ALP and bilirubin; fever unusual option B= emphysematous cholecystitis= suggests gallbladder necrosis or infection with gas forming bacteria; complication of untreated cholecystitis; patient not critically ill so unlikely option D= gallstone ileus option E= non-specific liver inflammation option F= acute pancreatitis

59 years woman -Nausea, chronic abdominal pain (1 year; worst after eating) -Hashimoto thyroiditis -Biopsy (corpus)= destruction of upper glandular layer of gastric mucosa and G-cell hyperplasia -Greatest risk for? a) gastric adenocarcinoma b) duodenal perforation c) curling ulcer d) aplastic anemia e) gastric MALT lymphoma

answer: gastric adenocarcinoma -Parietal cell destruction (upper glandular layer) + G cell hyperplasia + autoimmune disease (Hashimoto)= autoimmune atrophic gastritis (type A) -Can lead to intestinal metaplasia= increases risk of gastric adenocarcinoma -Increased risk of carcinoid tumor (autoantibodies against parietal cells which produce gastric acid)= achlorhydria= release of gastrin and G-cell hyperplasia= hypergastrinemia= enterochromaffin-like cell hyperplasia= carcinoid tumor option B= type B gastritis= H. pylori option D= macrocytic pernicious anemia due to intrinsic factor antibodies in gastritis type A option E= type B gastritis= H. pylori

56 years man -Upper abdominal pain (worsens after meals) -Urea breath test= positive -Endoscopy: diffuse mucosal atrophy and patchy erythema, but no ulcer -Diagnosis? a) gastric fundus b) distal esophagus c) gastric antrum d) duodenal bulb e) gastric pylorus

answer: gastric antrum -Chronic atrophic gastritis due to H. pylori= begins in gastrin antrum (pyloric antrum) and then spreads (to fundus or body)= increased risk of gastric MALToma -Biopsy= gastric antrum (most accurate) -Other type more common in body and fundus (autoimmune atrophic gastritis)= urea breath test is negative and associated with autoimmune diseases (e.g., pernicious anemia, vitiligo, autoimmune thyroiditis) -Atrophic gastritis= decreased gastric acid secretion (hypochlorhydria)= secondary increase in gastrin levels option A= autoimmune atrophic gastritis usually option B= indicated in patients with resistant reflux esophagitis; to rule out Barrett and dysplastic changes option D= ulcers associated with H. pylori, no ulcer here option E= indicated in patients with pyloric mass causing gastric outlet obstruction

55 years man -Intermittent epigastric pain -Improves with antacids but returns 2 hours after meals -Upper endoscopy: deep ulcer in posterior wall of duodenal bulb -Most likely to erode into which of the following structures? a) splenic vein b) pancreatic duct c) liver capsule d) descending aorta e) gastroduodenal artery f) transverse colon

answer: gastroduodenal artery Gastroduodenal artery runs directly posterior to first part of proximal duodenum (originates from common hepatic artery) -Supplies pylorus, proximal duodenum, head of pancreas option A= courses along posterior gastric fundus= damaged by gastric ulcers option B= distal duodenal ulcers; posterior to second part of duodenum option C= rare complication of duodenal ulcers; located superior to proximal duodenum option D= rare complication of gastric ulcers along the posterior gastric antrum option F= rare complication of gastric ulcers along greater curvature and anterior duodenal ulcers

61 years man -Epigastric pain, weight loss (6 months) -Low hemoglobin and MCV; occult stool positive -EGD: 2-cm raised lesion with central ulceration on lesser curvature of stomach -Histology? a) neutrophilic infiltration with pit abscesses b) foveolar and smooth muscle hyperplasia c) gland-forming columnar cells d) lymphocytic aggregates with noncaseating granulomas e) mucin-filled round cells

answer: gland-forming columnar cells -Epigastric pain + weight loss + microcytic anemia + lesion on lesser curvature= gastric adenocarcinoma) -Most common type of gastric cancer (90%) -Ulcer on lesser curvature= intestinal (type 1) gastric adenocarcinoma -Characterized by aggressive local spread and typically metastasizes to liver and regional lymph nodes -Risk factors: H. pylori infection, smoking, chronic gastritis option A= H. pylori gastritis; histology not consistent with H. pylori gastritis option B= chemical gastropathy; histology= edema of lamina propria; no high degree of weight loss and no suspicious ulcer; no chronic NSAIDs, alcohol or tobacco option D= Crohn disease option E= mucin filled round cells with peripheral nucleus= Signet ring cell carcinoma; weight loss, epigastric pain and bloody stool; endoscopy= diffuse involvement of stomach wall and linitis plastica, rather than single ulcer in lesser curvature

49 years man -Diarrhea and abdominal bloating (1 week) -Bulky, foul-smelling and difficult to flush stool -Recurrent dull epigastric pain (triggered by meals and lasts for a few days); tender but no rebound/guarding -Drinks alcohol daily -CT= pseudocyst -likely lined with? a) simple ductal epithelium b) calcified ductal cells c) granulation tissue d) pyogenic membrane e) columnar mucinous epithelium

answer: granulation tissue -Cystic lesion + chronic alcohol abuse + steatorrhea + recurrent epigastric pain= pancreatic pseudocyst due to chronic pancreatitis -Granulation tissue encapsulated pseudocyst, retaining enzyme-rich pancreatic fluids -Pseudocysts lack proper epithelial lining (in contrast to true cysts) -Due to disruption of pancreatic ductal system and extravasation of fluids (acute/chronic pancreatitis) -Most asymptomatic; large= mass effect= abdominal pain, biliary obstruction and potentially GI bleeds option A= pancreatic retention cyst; slow-growing, small, asymptomatic, benign lesions; large-cyst lesion found here with alcohol use= pseudocyst option B= strongly associated with chronic pancreatitis due to heavy alcohol use; signs of chronic pancreatitis= exocrine insufficiency (steatorrhea, fat malabsorption) and endocrine insufficiency (diabetes); wouldn't line cyst option D= abscesses, enclosing pus and bacteria; pancreatic abscesses due to acute pancreatitis, whereby leakage of pancreatic enzymes= surrounding tissue necrosis that is subsequently infected; pseudocysts can progress to abscesses but would present with fever, chills option E= cystadenoma of pancreas; benign tumor on body or tail; most are asymptomatic and slow-growing; large ones have mass effect and can transform to malignant lesion; not likely here due to CT

5 years girl -Severe headache, nausea, vomiting (6 hours) -Last week= fever, myalgias, sore throat (resolved with over-the-counter medication) -Bilateral optic disc swelling -Labs: hypoglycemia, high AST and ALT, elevated PT time -cause? a) autoimmune destruction of beta cells b) acute viral hepatitis c) hepatic mitochondrial injury d) bacterial meningitis e) ruptured berry aneurysm f) ethylene glycol poisoning

answer: hepatic mitochondrial injury -Reye syndrome caused by mitochondrial injury -In children, 3-5 days after aspirin treatment of a febrile viral illness (most commonly with VZV or influenza B) -Initially presents as hepatic encephalopathy (vomiting due to increased ICP (bilateral optic disc swelling), lethargy); progresses to delirium, seizures and coma -Other findings: hepatomegaly, elevated AST, ALT, hypoglycemia, elevated INR, metabolic acidosis -Liver biopsy (doubtful cases)= microvesicular steatosis option A= T1D; altered mental status, vomiting, diabetic ketoacidosis; wouldn't explain high transaminases and hyperglycemia not hypo option B= wouldn't explain bilateral optic disc swelling and hypoglycemia option D= fever, headache, vomiting, altered mental status; infection with N. meningitidis is the most common cause at this age group and may occur after pharyngitis or sinusitis; patient here doesn't have nuchal rigidity; positive Kernig sign (supine position; flex knee, extend thigh; if pain or resistance= positive) or positive Brudzinski sign (forced flexion of neck, causes flexion of knees and hips)) option E= subsequent subarachnoid hemorrhage; severe headache, impaired consciousness, bilateral optic disc swelling, fever, but not other signs here option F= ingestion antifreeze; altered mental status, vomiting and signs of hepatocellular injury; patients also present with sweet smelling breath, hallucinations, seizures, tetany, dyspnea, flank pain, hematuria due to calcium oxalate deposition in kidney

31 years man -Nausea, abdominal discomfort, scleral icterus -Six weeks ago= fever, joint pain, swollen lymph nodes, itchy rash on his trunk and extremities (persisted for 1 or 2 days); returned from Colombia 2 months ago -Fever, scleral icterus; infection with? a)Hepatitis B b)Enterotoxigenic E. coli c)Borrelia burgdorferi d)Hepatitis A e)Campylobacter jejuni

answer: hepatitis B -Medical history: serum sickness-like reaction (fever, arthritis, itchy rash) -Hepatitis B is associated with serum-sickness like reaction (occurs during prodromal period of the infection; mediated by immune complex formation)= usually resolve within a few weeks, but symptoms from acute HBV infection persist longer -other causes of serum sickness like reaction: antibiotics (cefaclor, penicillin), streptococcal infection and vaccines option B= Traveler's diarrhea option C= Lyme disease; fever, lymphadenopathy, transient arthritis, jaundice; would present with erythema migrans not itchy rash option D= not a known cause of serum sickness like reactions option E= bloody diarrhea with reactive arthritis; doesn't cause liver pathology; and missing classic symptoms of conjunctivitis and urethritis typically seen with reactive arthritis

31 years woman -Fever, RUQ pain, myalgia -Returned from a trip from Southeast Asia -Fever, jaundice and tender hepatomegaly -Lab: anti-hepatitis A IgM antibodies -Liver biopsy? a) hepatocytes with shrunken, eosinophilic cytoplasm and pyknotic nuclei b) acellular debris surrounded by lymphocytes and macrophages c) cystic spaces with scattered areas of cellular debris d) basophilic adipocytes remnants, filled with calcifications e) engorged hepatocytes with red blood cell infiltration

answer: hepatocytes with shrunken, eosinophilic cytoplasm and pyknotic nuclei -Hepatocytes with shrunken, eosinophilic cytoplasm and pyknotic nuclei= Councilman bodies -Councilman bodies are remnants of apoptotic hepatocytes (in acute hepatitis) -Apoptosis of virus-infected cells, such as HAV-infected hepatocytes, is induced by the release of granzyme B and perforin from cytotoxic T cells (extrinsic pathway of apoptosis) -Other signs of HAV= ballooning degeneration and bridging necrosis (confluent necrosis spanning adjacent lobules) option B= caseous necrosis; seen in TB, histoplasmosis, cryptococcosis.. option C= liquefactive necrosis; due to release of neutrophilic enzymes that degrade tissue into a liquid, viscous mass; in bacterial abscesses and brain infarcts option D= fat necrosis; due to release of lipases that degrade triglycerides, causing saponification; acute inflammation or trauma of tissue with a high concentration of adipocytes (e.g., pancreas, breast option E= hemorrhagic infarcts (red infarcts) of liver; in organs of dual blood supply (e.g., lung, GI tract, testes, and liver); secondary to occlusion of vessels and subsequent extravasation of RBCs into ischemic tissue

65 years man -Family history of colon cancer -Colonoscopy: 4mm polyp in upper sigmoid -Biopsy: which one associated with lowest potential for malignant transformation? a) branching tubules embedded in lamina propria b) tree-like branchings of muscularis mucosa c) regenerating epithelium with inflammatory infiltrate d) finger-like projections with a fibrovascular core e) hyperplastic epithelium at the base of crypts

answer: hyperplastic epithelium at the base of crypts -Hyperplastic polyps consist of hyperplastic epithelium at base of the crypts of Lieberkühn -Often in rectosigmoid and left colon -Benign= low risk of transforming into malignancy -Other examples of polyps that are benign: mucosal polyps and submucosal polyps option A= tubular adenoma; small risk of malignant transformation= 5%) option B= Peutz-Jeghers syndrome; multiple non-neoplastic hamartomous polyps; PJS= increased risk= 40%) option C= UC= pseudopolyps= low risk of malignant transformation option D= Villous adenoma= highest risk= 50%)

3 days girl -Jaundice after birth; normal hemoglobin; high bilirubin (direct normal) -Infant dies; brain= deep yellow staining of the basal ganglia and subthalamic nuclei bilaterally -Cause? a) defective intracellular bilirubin transport b) increased degradation of red blood cells c) extrahepatic obliteration of the biliary tree d) impaired glucuronidation of bilirubin e) decreased bilirubin uptake in hepatocytes

answer: impaired glucuronidation of bilirubin -impaired glucuronidation= elevated unconjugated bilirubin -Pathological neonatal jaundice, normal conjugated bilirubin levels, absence of anemia= Type 1 Crigler-Najjar syndrome (in which unconjugated bilirubin crosses the BBB, leading to kernicterus and potentially death) option A= Dubin Johnson and Rotor syndrome= conjugated hyperbilirubinemia and milder conditions option B= hemolytic anemia; unconjugated hyperbilirubinemia; but no anemia here option C= biliary atresia; jaundice, pale stool and dark urine (cholestasis), elevated conjugated bilirubin too option E= Gilbert syndrome; also decreased conjugation= unconjugated hyperbilirubinemia; rarely present at birth and mild; jaundice in stress

2 weeks infant -4 episodes of bilious vomiting and inconsolable crying (3 hours) -Upper GI contrast= duodenojejunal junction to right of vertebral midline -Air-filled cecum is noted in RUQ -Cause? a)Defective neural crest migration b)Failure of duodenal recanalization c)Nonrotation of the intestines d)Incomplete intestinal rotation e)Arrested rotation of ventral pancreatic bud f)Hypertrophy and hyperplasia of the pyloric sphincter g)Resorption of a small bowel segment

answer: incomplete intestinal rotation -Duodenojejunal junction right of midline and cecum in RUQ= incomplete intestinal rotation -Bilious vomiting without abdominal distention -Obstruction due to extrinsic duodenal compression by peritoneal bands (Ladd bands) that cross over duodenum to fix the abnormal cecum to the liver or peritoneum option A= Hirschsprung disease option B= duodenal atresia; associated with down syndrome and maternal polyhydramnios option C= cecum in LLQ and small bowel in right side option E= annular pancreas; results in extrinsic compression of duodenum, usually proximal to major duodenal papilla; present with nonbilious vomiting and abdominal distention; upper GI series: eccentric or concentric narrowing of the 2nd portion of duodenum, double bubble sign, symmetrical dilation of proximal duodenum; associated with maternal polyhydramnios option F= hypertrophic pyloric stenosis option G= jejunal and ileal atresia; caused by an in-utero vascular disruption that leads to ischemic necrosis of fetal intestine; necrotic tissue is resorbed, leaving blind proximal and distal ends; bilious vomiting; also distention of upper abdomen, delayed passage of meconium and triple bubble sign; associated with maternal polyhydramnios and trisomy 21 note (midgut volvulus): ladd bands attach cecum to retroperitoneum

47 years man -Cirrhosis; erythema over thenar and hypothenar eminences -Numerous blanching lesions over trunk and upper extremities that have a central red vessel with thin extensions radiating outwards -Cause? a) increased circulating ammonia b) decreased circulating albumin c) decreased circulating thrombopoietin d) decreased circulating testosterone e) increased circulating cortisol f) increased circulating estrogen

answer: increased circulating estrogen -Palmar erythema + spider angiomas (also gynecomastia, testicular atrophy, decreased body hair)= excess estrogen -Hyperestrogenism with liver dysfunction= due to impaired hepatic metabolism of estrogen and androstenedione (converted to estrogen by aromatase in adipose cells) option A= hepatic encephalopathy option B= peripheral edema and ascites option C= together with congestive splenomegaly with platelet sequestration (portal hypertension)= thrombocytopenia option D= (in men, reduced libido, erectile dysfunction, infertility, testicular atrophy option E= Cushing syndrome; purple abdominal striae, flushing of face

71 years woman -Intermittent abdominal pain, vomiting and obstipation -Multiple episodes of upper abdominal pain (past year) -Distended abdomen with diffuse tenderness and high-pitched bowel sounds -X-ray- dilated bowel, multiple air-fluid levels and gas in biliary tree -Cause? a) perforation of peptic ulcer b) inflammation of gallbladder wall c) obstruction of the common bile duct d) occlusion of the superior mesenteric artery e) torsion of the large intestine

answer: inflammation of the gallbladder wall -Abdominal distention, obstipation, vomiting, high pitched bowel sounds and dilated bowel loops (GI obstruction) and pneumobilia= gallstone ileus -Complication of chronic cholecystitis Cholecystoenteric fistula= gallstone dislodges to distal ileum (narrow) option A= paralytic ileus could explain X-ray; but air would be in peritoneum (peritonitis); also, paralytic ileus= absent or decreased bowel sounds option C= would present with jaundice symptoms option D= cause bowel infarction= peritonitis, paralytic ileus= air in peritoneum and decreased bowel sounds option E= volvulus; can lead to obstruction; rarely can present with radiolucency in the right infra-diaphragmatic region (Chilaiditi sign)

47 years man -GERD; severe heartburn and belching -EGD= erythema and erosions in the distal esophagus -Mechanism of drug for treatment? a) enhancement of the mucosal barrier b) inhibition of D2 receptors c) neutralization of gastric acid d) inhibition of H2 receptors e) inhibition of ATPase

answer: inhibition of ATPase -PPIs= irreversibly inhibit H+/K+ ATPase in parietal cells of stomach and reduce ATP dependent secretion of H+ into gastric lumen -First-line treatment for GERD option A= sucralfate; forms protective barrier= acts as acid buffer, and promotes HCO3 production= commonly used in PUD, not GERD option B= a)metoclopramide; increases LES tone and increases gastric motility; can improve GERD, not used due to side effect (parkinsonism) option C= antacids, such as calcium carbonate; used for mild intermittent heartburn and/or GERD; however, this patient has severe symptoms option D= ranitidine; inhibit histamine-dependent gastric acid secretion; antagonize H2 receptors on parietal cells; used for mild GERD not severe

75 years man -Mass in the anal canal proximal to the pectinate line -Primary metastasis to which lymph nodes? a) inferior mesenteric b) internal iliac c) superior mesenteric d) external iliac e) para-aortic f) superficial inguinal g) deep inguinal

answer: internal iliac -Internal iliac= drain upper part of anal canal -They also drain= gluteal region, inferior pelvic viscera (including rectum, prostate, part of cervix), perineum (including membranous urethra and spongy urethra) option A= splenic flexure to upper rectum option C= distal duodenum to splenic flexure; also receive drainage from inferior mesenteric lymph nodes option D= uterus and superior portion of bladder option E= testes, ovaries, uterus, kidneys and adrenals option F= lower part of anal canal, skin below umbilicus and parts of genitourinary system option G= glans penis, vulva, in some cases anus distal to pectinate line

45 years woman -Mild RUQ pain (3 months); no chills, fever or weight loss -Medications= transdermal estrogen (menopause) -US= well-demarcated, homogenous, hyperechoic mass surrounded by normal liver tissue -Biopsy would put patient at greatest risk for? a) intraperitoneal hemorrhage b) biliary peritonitis c) bactermia d) metastatic spread e) anaphylactic shock

answer: intraperitoneal hemorrhage -Hepatic hemangioma (cavernous vascular spaces of variable size lined by flat endothelial cells) -most common in women (30-50 years); become enlarged during hormonal therapy -Most asymptomatic, diagnosed incidentally on imaging, appear hyperechogenic (due to increased vascularity), well-demarcated, homogenous -Highly vascular tumor= biopsy= risk of severe hemorrhage option B= risk for any liver biopsy; extremely rare option C= biopsy of intrahepatic abscess= risk of spreading infection; US= poorly demarcated, fluid-filled, hypoechoic lesions surrounded by hyperechoic rim (edema and hyperemia option D= risk of HCC; seed tumor cells throughout track of biopsy needle= risk of metastases; US= ill-defined mass with irregular borders and variable echogenicity with areas of necrosis and calcification option E= biopsy of hepatic hydatid cyst= anaphylactic shock (highly antigenic hydatid fluid); US= unilocular, anechoic, well-defined cyst with hyperdense internal septa (spoke-wheel pattern), usually with daughter cysts floating within

newborn -Bilious projectile vomiting -Pregnancy: complicated with polyhydramnios; mother smoked during pregnancy -Distended upper abdomen -X-ray: 3 distinct, localized gas collections in upper abdomen and gasless distal abdomen -Diagnosis? a) necrotizing enterocolitis b) duodenal atresia c) meconium ileus d) malrotation with volvulus e) hypertrophic pyloric stenosis f) jejunal atresia g) hirschsprung disease

answer: jejunal atresia -Distended abdomen and bilious emesis= obstruction distal to pylorus -Triple bubble sign: jejunal atresia -Cause: vascular disruption of the mesenteric blood vessels in utero, resulting in necrosis and segmental reabsorption of intestine -Residual small bowel distal to atresia= wind around stalk of the ileocolic artery in spirals (apple peel atresia) -Risks: maternal use of cocaine, MDMA, cigarettes option A= pneumatosis intestinalis; diarrhea, rectal bleeding, abdominal tenderness are all absent; most common in premature infants option B= presents same, but double bubble sign; association with down syndrome option C= no failure to pass meconium; usually due to ileal obstruction from impacted stool; associated with cystic fibrosis option D= present same without polyhydramnios; hemodynamic instability, or signs of peritonitis; X-ray: double bubble sign (duodenal obstruction) or pneumoperitoneum (perforation) option E= nonbilious vomiting; olive shaped mass; sunken fontanelle option G= abdominal distention, bilious vomiting, failure to pass meconium; narrowing at rectosigmoid region

45 years man -Burning epigastric pain (worse with meals) -High pulse, low BP= dies -Erosion of the right gastric artery -Perforation location? a) anterior duodenum b) posterior duodenum c) fundus of stomach d) greater curvature of stomach e) lesser curvature of stomach

answer: lesser curvature of stomach Lesser curvature= right (common hepatic artery) and left gastric arteries (celiac trunk) option A= superior pancreaticoduodenal artery/anterior branches option B= a)gastroduodenal artery (common hepatic artery) option C= short gastric arteries (splenic artery branches) option D= left and right gastroepiploic arteries

25 years woman -Poorly controlled asthma; mother died due to emphysema -Labs: decreased levels of protease inhibitor that inhibits elastin degradation -Labelled organ involved in pathogenesis? a) stomach b) liver c) psoas major muscle d) aorta e) left kidney f) spleen

answer: liver -alpha-1 antitrypsin (serine protease that inhibits elastin degradation) -Mutations of AAT gene leads to production of misfolded protein within hepatocytes= defective AAT can't be secreted and aggregates= hepatic damage and possibly cirrhosis and/or primary liver cancer -Histology: protein aggregations as PAS-positive, spheric inclusion bodies in periportal hepatocytes

15 years girl -Abdominal pain, nausea, vomiting, diarrhea, decreased appetite (2 days) -Slight fever; abdominal tenderness to palpation with guarding in the RLQ -Leukocytosis -Most likely underlying cause? a)Bacterial mesenteric lymphadenitis b)Pseudomembranous plaque formation in the colon c)Diverticular inflammation d)Congenital anomaly of the omphalomesenteric duct e)Lymphatic tissue hyperplasia f)Gestation in the fallopian tube

answer: lymphatic tissue hyperplasia -Acute appendicitis -Cause (most common): lymphatic tissue hyperplasia (children), fecalith obstruction (adults) -Obstruction= mucus and fluids stasis= bacterial overgrowth (inflammation of organ) -Present with= abdominal pain (dull migratory periumbilical pain that progresses to sharp RLQ pain), fever, loss of appetite, nausea, and leukocytosis option A= pseudoappendicitis; RLQ pain, fever, vomiting, leukocytosis; primarily affects children under age of 5 option B= a)Pseudomembranous colitis; C. difficile) Diverticular inflammation (low-grade fevers, loss of appetite, diarrhea, vomiting, and LLQ pain; occurs in patients above 60 years option C= Meckel's diverticulum; can result in abdominal pain, nausea, vomiting BUT usually asymptomatic; symptomatic would present with painless GI bleeding; symptoms within first 2 years of life option F= ectopic pregnancy; lower unilateral abdominal pain; guarding, vomiting, loss of appetite; become symptomatic about 7-8 weeks after last menses

44 years man -Fatigue and increased straining on defecation -Weight loss, family history of colon cancer (maternal uncle and grandfather) (mother died or ovarian cancer at 46) -Conjunctival pallor; low hemoglobin and hematocrit, and MCV -Colonoscopy: exophytic mass in ascending colon -Pathology: poorly differentiated adenocarcinoma -Genetics: mutation in the MSH2 gene -Diagnosis? a) familial adenomatous polyposis b) li-fraumeni syndrome c) turcot syndrome d) peutz-jeghers syndrome e) gardner syndrome f) lynch syndrome

answer: lynch syndrome -Lynch syndrome= autosomal recessive mutation in DNA mismatch repair genes (MSH2, MLH1); genetics best way to diagnose -Risk: 80% of colorectal cancer (proximal colon) -Can use 3-2-1 rule à lynch syndrome associated cancer in > 3 family members, within 2 generations, and in 1 relative under age of 50 years option A= autosomal dominant; APC mutation; 100% risk by 45 years option B= autosomal dominant; mutation in p53; subsequent loss of heterozygosity results in development of multiple malignancies (e.g., sarcomas, adrenal gland, breast cancer, leukemia) at early ages option C= variant of FAP or lynch syndrome, associated with increased risk of colorectal cancer as well as malignant CNS tumors (e.g., medulloblastoma with FAP, glioma with lynch); MSH2 can be found; lack of evidence of CNS tumor= unlikely option D= autosomal dominant; STK11 mutation; manifests with multiple hamartomatous polyps (predominantly jejunum) and mucocutaneous hyperpigmentation of lips, buccal mucosa, palms, and soles; increased risk of colorectal cancer and others option E= subtype of FAP; autosomal dominant mutation in APC; presence of hundreds of colonic polyps + bony and/or soft tissue tumors (e.g., osteoma, desmoid tumors, lipomas, fibromas); development of colorectal cancer= 100%

71 years woman -Fatigue, anorexia, abdominal swelling, shortness of breath, weight loss (2 months) -Jaundice, bilateral temporalis muscle wasting, hepatosplenomegaly, tense ascites -US: multiple hepatic masses and enlargement of portal vein -Cause? a) metastatic spread of malignant cells from colon b) Malignant transformation of hepatocytes c) proliferation of hepatic capillaries d) hyperplasia of atypical bile duct tissue e) lymphoproliferative disorder of hepatic sinusoids

answer: metastatic spread of malignant cells from colon -Metastatic liver disease (most common) -Muscle wasting (cachexia), hepatosplenomegaly, portal hypertension (dilation of portal vein, ascites) -US: multiple masses -Main cause: colorectal carcinoma option B= HCC, usually single mass with irregular border option C= hepatic hemangioma option D= cholangiocarcinoma option E= hepatosplenic T cell lymphoma

46 years woman -Diarrhea and abdominal pain -Returned from Egypt -Tenderness in RLQ; occult blood; unicellular organisms with engulfed erythrocytes -Treatment? a) doxycycline b) metronidazole c) albendazole d) paromomycin e) praziquantel f) ciprofloxacin

answer: metronidazole -Metronidazole= nitroimidazole antibiotic for infections with anaerobic bacteria and certain protozoal infection -Treatment of choice for amebic dysentery and amebic liver abscess (unicellular organisms with engulfed erythrocyte) -Tinidazole effective alternative -Afterwards intraluminal amebicide= paromomycin or iodoquinol to eliminate intestinal carriage of E. histolytica option A= cholera; voluminous rice water stools; dehydrated, febrile option C= anthelminthic (nematodes and cestodes like cysticercosis or echinococcosis); absence of eggs and larvae= no intestinal helminthic infection option D= for carrier state option E= trematode infections like schistosomiasis, clonorchiasis and certain cestodes (taeniasis); presence of eggs in feces option F= dysentery caused by C. jejuni or Yersinia enterocolitica; can present same but do not ingest erythrocytes

54 years man -Yellow discoloration of skin -Works as safety inspector at local nuclear power plant -Not physically active; high calories diet; drink alcohol daily -High BMI; jaundice, scleral icterus (scratch marks on extremities) -Nontender mass below right costal margin -Cause? a) infection b) lithiasis) c) hemolysis d) steatosis e) neoplasia f) autoimmune disease

answer: neoplasia -Neoplasia of GI tract= resulted in biliary system obstruction (pruritus, loss of appetite, Courvoisier sign (enlarged gallbladder and painless jaundice) -Possible etiologies: pancreatic head carcinoma, cholangiocarcinoma and HCC -Exposure to radiation (working at power plant) increase risk of malignancy; alcohol=HCC and pancreatic cancer; obesity= pancreatic adenocarcinoma -Diagnosis: LFTS, parameters of cholestasis, tumor markers (AFP, CA19-9, CEA) option A= no fever, nausea, vomiting and abdominal pain option B= cholelithiasis= no biliary colic; choledocholithiasis= no RUQ pain, nausea, vomiting, pale stool, dark urine option C= no anemia, fatigue, weakness or pallor option D= secondary to alcohol use or body habitus (NASH)= jaundice; generally asymptomatic option F= AIH, PBC, PSC; PSC more common in men; fatigue, RUQ pain

42 years man -Intermittent fever, abdominal pain, bloody diarrhea, weight loss -Colonoscopy: circumferential erythematous lesions without interruption from anal verge to cecum -Biopsy: mucosal and submucosal inflammation with crypt abscesses -Colon cancer characteristics? a) unifocal lesion b) late p53 mutation c) non-polypod dysplasia d) low grade lesion e) early APC mutation

answer: non-polypoid dysplasia -Ulcerative colitis -Recurrent cycles of inflammation à release of inflammatory markers= promote cellular hyperplasia= ultimately non-polypod dysplasia (reversible; but can lead to neoplasia) option A= sporadic colorectal cancer; UC cancer= multifocal lesions option B= sporadic colorectal cancer; UC cancer= early p53 mutation option D= UC cancer= high grade lesion option E= sporadic colorectal cancer; UC cancer= uncommon or late in course

6 years boy -Mild episodic epigastric pain (every 1-2 months; lasts few hours); radiates to the back; mild nausea -Sister= congenital heart disease -Otherwise, healthy; and met all developmental milestones -Explanation? a) duodenal atresia b) hypertrophic pyloric stenosis c) biliary cyst d) tracheoesophageal fistula e) intestinal malrotation f) pancreas divisum

answer: pancreas divisum -Pancreas divisum= congenital malformation= failure of fusion of ventral and dorsal pancreatic ducts (8 weeks of gestation; 10% of population; usually asymptomatic, some chronic abdominal pain or recurrent episodes of acute pancreatitis) -Diagnosed by endoscopic ultrasonography or MRCP= ventral pancreatic duct that drains along with CBD into major duodenal papilla and dorsal pancreatic duct that drains separately into the duodenum via minor duodenal papilla option A= delayed meconium passage, bilious vomiting, epigastric distention, scaphoid lower abdomen option B= postprandial, nonbilious projectile vomiting, visible epigastric peristaltic waves, palpable olive shaped structure option C= causes recurrent acute pancreatitis; due to pancreatic duct obstruction and pancreaticobiliary reflux; would have obstructive jaundice, palpable right hypochondrial mass option D= manifests immediately after birth (following first feeding) with features of aspiration (e.g., choking, coughing, cyanosis) option E= recurrent episodes of abdominal pain as a result of chronic midgut volvulus or chronic duodenal obstruction; volvulus=manifestations of malabsorption syndrome (e.g., diarrhea, failure to thrive) and chronic duodenal obstruction= episodes of bilious vomiting

47 years woman -Chronic epigastric pain -Loose, foul-smelling stools, weight loss; alcohol daily (25 years) -Stool: pale, loose and fecal fat content is elevated -IgA serum anti-tissue transglutaminase antibody assay= negative -Further evaluation? a) inflammation of subcutaneous fat b) trophozoites on stool microscopy c) pancreatic calcifications d) villous atropy of duodenal mucosa e) positive lactulose breath test

answer: pancreatic calcifications -Chronic epigastric pain + alcohol use + features of pancreatic insufficiency (steatorrhea, weight loss)= chronic pancreatitis -CT= pancreatic calcifications, ductal strictures, and ductal dilation (Chain of lakes in pancreatic duct) -Pancreatic function tests (e.g., fecal elastase-1 measurement, 72-hour quantitative fecal fat test) can assess degree of pancreatic exocrine and enzyme deficiency option A= erythema nodosum (inflammation of subcutaneous fat) is associated with UC= bloody diarrhea not steatorrhea option B= Giardiasis; manifests with steatorrhea and weight loss; usually in people who travel to resource-limited areas with poor sanitation option D= celiac disease option E= Small intestinal bacterial overgrowth; manifest with chronic abdominal pain, steatorrhea, and weight loss; epigastric pain wouldn't be expected; SIBO is caused by anatomical abnormalities (adhesions or strictures, small intestinal diverticulosis) or motility disorders (e.g., IBS, radiation enteritis or scleroderma)

58 years man -Abdominal pain, vomiting (4 days) -Initially non-bilious, now bilious -Similar episodes (past 6 years); smoker (25 years); alcohol daily -Epigastric mass -Diagnosis? a) chronic cholecystitis b) retroperitoneal fibrosis c) hypertrophic pyloric stenosis d) pancreatic pseudocyst e) gastric adenocarcinoma

answer: pancreatic pseudocyst -Complication of acute or chronic pancreatitis -Cause: leakage of pancreatic exocrine secretions form damaged ducts -Recurrent epigastric pain, vomiting, chronic alcohol consumption= pancreatitis likely -Pancreatic pseudocysts when large enough= palpable epigastric mass and bilious vomiting (due to extrinsic compression of the distal duodenum) or non-bilious vomiting (gastric outlet obstruction) -Imaging: well-defined fluid collection in vicinity of pancreas option A= consequence of recurrent acute cholecystitis; upper abdominal pain and bilious or non-bilious vomiting; however, gallbladder is shrunken and fibrosed; wouldn't explain mass option B= recurrent abdominal pain (hydronephrosis or mesenteric ischemia) and vomiting (hydronephrosis and uremia); typically, due to hydronephrosis (mass in flanks); also, lower limb edema, varicocele, claudication option C= non-bilious vomiting option E= vomiting is a late manifestation; non-bilious)

45 years woman -Hypothyroidism; fatigue, epigastric pain (after food) -Pale conjunctivae; labs: decreased hemoglobin, increased serum gastrin -EGD: inflammation of gastric body and fundus -Biopsy (antrum): G cell hyperplasia (shown in answer) -Cause? a) S cell hyperplasia b) mucosal cell hyperplasia c) enterochromaffin-like cell hyperplasia d) parietal cell destruction e) chief cells destruction f) I cell destruction

answer: parietal cell destruction -Biopsy shows G cell hyperplasia -Parietal cell destruction in fundus= type A gastritis= reduced gastric acid production -Reduced acid= G cell hyperplasia in antrum and hypergastrinemia -Also impairs intrinsic factor production= decreased B12 absorption= pernicious anemia -Iron deficiency anemia (earlier in disease course)= due to decreased solubility of iron in setting of hypochlorhydria option A= causes excess secretin; increases pancreatic bicarbonate secretion and decreases gastric acid secretion (inhibits gastrin secretion option B= leads to increased secretion of bicarbonate, which increases gastric pH, hypochlorhydria can lead to microcytic anemia; but wouldn't explain inflammation of body and fundus, elevated gastrin, or G cell hyperplasia option C= increases histamine and subsequently gastric acid production, which can lead to inflammation of body and fundus. Gastrin stimulates histamine release from ECL cells, but wouldn't occur as a consequence of ECL hyperplasia; combination of anemia, hypergastrinemia, and G cell hyperplasia= less likely option E= in chronic gastritis with mucosal atrophy= impaired protein digestion due to decreased production of pepsinogen; pepsin cleaves B12 from food proteins= macrocytic anemia (no cleaving proteins); wouldn't affect gastrin production and G cell hyperplasia option F= decreased cholecystokinin; increases pancreatic secretion, mediates gallbladder contraction; inhibits gastric emptying; wouldn't explain anemia, inflammation of body and fundus, elevated gastrin or G cell hyperplasia

52 years woman -5-hour history of diffuse, severe abdominal pain, nausea and vomiting -Epigastric pain, improved with eating -Takes ibuprofen for osteoarthritis (2-year history), fever, tachycardia -Abdominal tenderness and guarding, bowel sounds decreased -X-ray: free air below diaphragm -Cause of symptoms? a) perforated peptic ulcer b) acute pancreatitis c) acute mesenteric ischemia d) gastric cancer e) gastroesophageal reflux disease f) cholecystolithiasis

answer: perforated peptic ulcer -Perforation of peptic ulcer= acute, diffuse abdominal pain, nausea, vomiting, fever, and tachycardia + pneumoperitoneum (free air below diaphragm) -Secondary peritonitis (abdominal guarding) and developing paralytic ileus (decreased bowel sounds) -Duodenal ulcer here= relieved by food + ibuprofen (most common in anterior duodenal ulcer) option B= everything here; except it worsens after meals and Chest X-ray findings option C= everything same except X-ray findings (normal in early stages and distended loops and small gas enclosures in wall of intestine (indicating transmural ischemia) in later stages option D= no weight loss and X-ray= less likely option E= asymptomatic; biliary colic with food intake

57 years woman -Epigastric pain, improves with meals -Endoscopy= 0.5cm mucosal breach in the anterior duodenal bulb that extends into submucosa -Biopsy: hypertrophy of Brunner glands -Greatest risk? a) perforation b) hematemesis c) MALT lymphoma d) gastric outlet obstruction e) adenocarcinoma f) pernicious anemia

answer: perforation -Second most common complication of chronic PUD, bleeding being first -Most common location for ulcer perforation: anterior duodenum -Posterior ulcers: massive bleeding (hematemesis) from adjacent gastroduodenal artery option B= bleeding most common complication regardless of location; but most commonly caused by posterior duodenal ulcer; anterior ulcer; less likely to produce massive bleeding which presents with hematemesis option C= chronic H. pylori= mostly duodenal ulcers; but MALT lymphomas localized in stomach, rare in duodenum option D= malignancy; duodenal ulcer can cause this, but less likely option E= rare complication of chronic gastric ulcers; duodenal ulcers are benign option F= autoimmune gastritis

46 years man -Jaundice, pruritus; diagnosed with ulcerative colitis (pANCA positive) -High bilirubin; high direct bilirubin; high ALP -MRCP= intrahepatic bile duct strictures alternating with areas of dilation -Biopsy? a) periductal concentric scarring and fibrosis b) Periportal lymphocytic infiltration and piecemeal necrosis c) Diffuse fibrosis with PAS-staining globules d) Irregular ductal glands lined by atypical and pleomorphic cells e) Ductopenia and fibrotic degeneration of periportal hepatocytes

answer: periductal concentric scarring and fibrosis -jaundice, pruritus, UC (in 90%) + pANCA (positive in UC and PSC) LFTs + MRCP + man= PSC -Biopsy= periductal concentric scarring and fibrosis -Increased risk of development of gallbladder cancer and Cholangiosarcoma (10-15%) option B= chronic viral hepatitis or autoimmune hepatitis option C= AATD option D= intrahepatic cholangiocarcinoma option E= PBC

49 years man -HIV; mild diffuse tenderness throughout lower quadrants -CD4+ T cell count is 180/mm3 -Colonoscopy: multiple hemorrhagic nodules in rectum and descending colon -PCR= positive for HHV-8 -Histology? a) cords of atypical cells with extracellular mucin b) enlarged cells with ground-glass nuclei and central clearing c) enlarged cells with intranuclear inclusion bodies d) polygonal cells with racket-shaped organelles e) spindle-shaped cells with lymphocytic infiltration f) mucin-filled cell with peripheral nucleus

answer: spindle-shaped cells with lymphocytic infiltration -HIV + low CD4+ (<200/mm3) + HHV8= Kaposi sarcoma -Histology: spindle-shaped cells with lymphocytic infiltration -Kaposi sarcomas originate from endothelial cells and usually manifest as asymptomatic vascular tumors of the skin -Can also involve lymph nodes and GI tract (diarrhea and abdominal pain) -Additional histology: slit-like vascular spaces and extravasated erythrocytes option A= colorectal mucinous adenocarcinoma option B= Orphan Annie eye nuclear inclusions, characteristic of papillary thyroid carcinoma and autoimmune thyroiditis (Hashimoto) option C= CMV colitis; endoscopy= linear ulcers; CMV infection with HIV presents when CD4+ T cells <50/mm3 option D= Birbeck granules, seen in Langerhans cell histiocytosis; rarely involves GI option F= signet ring cell carcinoma of stomach

58 years woman -Gradual swelling on right side of face; smoker (18 years) -Right-sided, movable facial mass (nontender); protrusion of the right lateral wall of oropharynx -No cervical lymphadenopathy; cranial nerves intact -MRI= 3.3cm sharply demarcated, lobulated hyperintense mass arising from right parotid -Lesion surrounded by hypointense capsule -Biopsy: nests of epithelial cells mixed with a chondromyxoid stroma -Diagnosis? a)Mucoepidermoid carcinoma b)Pleomorphic adenoma c)Papillary cystadenoma lymphomatosum d)Adenoid cystic carcinoma e)Granulomatous sialadenitis

answer: pleomorphic adenoma -Gradually growing, painless, mobile, well-encapsulated unilateral mass from parotid without facial nerve palsy= pleomorphic adenoma (most common benign tumor) -Admixture of epithelial and chondromyxoid stroma= pathognomic -Complete resection recommended= can become malignant option A= most common malignant salivary gland tumor; gradually growing, painless unilateral mass from parotid; usually invade surrounding tissue and immobile, along with cervical lymphadenopathy, facial nerve palsy, and/or trismus (can't open jaw); biopsy= mixture of mucin producing columnar cells and epidermoid cells option C= Warthin tumor; in smokers; gradually growing, painless, mobile, and well-encapsulated mass from parotid without facial nerve palsy; biopsy= cystic components, abundant lymphocytes, germinal centers, and lymph node like stroma option D= gradually growing, painless unilateral mass arising from parotid; invade surrounding tissue, immobile with cervical lymphadenopathy, facial nerve palsy, and/or trismus; biopsy= admixture of ductal and myoepithelial cells displaying cribriform, tubular, or solid-growth patterns option E= painless and mobile parotid swelling without facial nerve palsy; noncaseating granulomas in biopsy

40 years woman -Anal pain (during defecation); lasts several hours -Bright red blood on toilet paper; 3 bowel movements per week -Longitudinal, perianal tear -Injury to? a) posterior midline of the anal canal, distal to the pectinate line b) anterior midline of the anal canal, proximal to the pectinate line c) anterior midline of the anal canal, distal to the pectinate line d) posterior midline of the anal canal, proximal to the pectinate line e) lateral aspect of the anal canal, distal to the pectinate line

answer: posterior midline of the anal canal, distal to the pectinate line -90% of anal fissures occur at the posterior midline of anal canal, distal to pectinate (dentate) line -Etiology: benign and primary in nature (most common is local trauma, e.g., passing of hard stools due to constipation), persistent diarrhea, vaginal delivery, or anal sex -Predilection for posterior midline= poor perfusion to this area -Often, a skin tag (sentinel pile) formed by hypertrophied papillae would be visible in the perianal region at site of fissure option B= common location of internal hemorrhoid and adenocarcinoma of anal canal; internal hemorrhoids= passage of bright red blood per rectum with a bowel movement; defecation is PAINLESS option C= a)10% of anal fissures; secondary to anal surgery, IBD, infectious (syphilis) or malignancy (leukemia) option D= internal hemorrhoids option E= external hemorrhoids, are typically painful and can cause per rectal bleeding; would be visible as perianal swellings on physical examination; less than 1% of secondary anal fissures here

12 years girl -Severe epigastric pain, nausea, vomiting (2 hours) -Father: history of similar episodes of abdominal pain and developed diabetes mellitus at age of 30 -Abdomen: rigid and guarding; US= diffuse enlargement of pancreas; no gallstones -Cause? a)Defective bilirubin glucuronidation b)Elevated serum amylase levels c)Increased beta-glucuronidase activity d)Premature activation of trypsinogen e)Defective elastase inhibitor f)Impaired cellular copper transport

answer: premature activation of trypsinogen -Severe epigastric pain + diffuse enlargement of pancreas= acute pancreatitis (usually occurs with gallstones or alcohol use); absence of risk factors + atypical age of presentation + family history= hereditary component -Premature activation of trypsinogen to trypsin= activates pancreatic digestive enzymes (lipase, amylase and protease)= cause pancreatic autodigestion and inflammation -This is underlying mechanism, regardless of etiology (gallstones, alcohol) -Hereditary pancreatitis= mutated PRSS1 gene= promotes intrapancreatic trypsinogen activation option A= Gilbert and Crigler-Najjar; unconjugated hyperbilirubinemia; black brown pigment stones= risk factor for acute pancreatitis; this patient has no jaundice or gallstones option B= elevated serum amylase more than 3 times + acute epigastric pain= acute pancreatitis; result of not cause of option C= result of bacterial infections in biliary tract; it deconjugates direct bilirubin= excessive unconjugated bilirubin= brown pigment stones= can cause acute pancreatitis; no stones in this patient option E= alpha-1 antitrypsin deficiency= liver cirrhosis and lung emphysema, doesn't affect pancreas option F= Wilson disease; presents with liver disease (e.g., jaundice, hepatomegaly, ascites) in early childhood and neurologic manifestations (e.g., dysarthria, ataxia, parkinsonism) in late childhood/adulthood

50 years man -upper abdominal pain, nausea, vomiting, diarrhea (6 months); weight loss -Bilateral pitting pedal edema -Endoscopy: prominent rugae in gastric fundus -Biopsy: parietal cell atrophy -Cause? a)serotonin-secreting gastric tumor b)Proliferation of gastric mucus producing cells c)Neoplasia of submucosal lymphoid tissue d)Chronic H. pylori infection e)Excessive somatostatin secretion f)ectopic secretion of gastrin

answer: proliferation of gastric mucus producing cells -Bilateral pitting pedal edema (loss of protein) + prominent rugae in the fundus + parietal cell atrophy= Menetrier disease (proliferation of gastric mucus producing cells) -Pathogenesis= increased signaling of EGFR= which results in proliferation of epithelial cells of mucous cell compartment option A= Carcinoid tumor option C= gastric lymphoma; pain, weight loss; no edema; endoscopy: thickened rugae; histology= abnormal lymphoid tissue not parietal atrophygastric lymphoma; pain, weight loss; no edema; endoscopy: thickened rugae; histology= abnormal lymphoid tissue not parietal atrophy option D= no edema; antral region; ulcers option E= Somatostatinoma; weight loss and pain; triad of glucose intolerance; gallstones and diarrhea/steatorrhea; in stomach= submucosal mass; histology= densely granular cells in rosette pattern= common finding of neuroendocrine tumors option F= ZES; therapy-resistant PUD (in duodenum) and diarrhea, weight loss; endoscopy: prominent gastric folds; biopsy: PARIETAL CELL HYPERPLASIA

3 days newborn -Abdominal distention, crying, bilious vomiting -Not passed meconium; abdominal distention, tight anal sphincter -Explosive passage of air and feces on removal of examining finger -Abnormal development of? a) Muscularis propria and adventitia b) Muscularis mucosae and serosa c) Epithelium and lamina propria d) Submucosa and muscularis externa e) Epithelium and submucosa f) Muscularis mucosae and lamina propria

answer: submucosa and muscularis externa -Hirschsprung disease -Meissner (submucous) plexus + Auerbach (myenteric) plexus --> absent at anorectal line -Cause: defective migration of plexus ganglion cell precursors (neural crests) to distal colon -Peristalsis is uncoordinated, motility is slow, relaxation is impeded --> excessive contraction of intestinal muscles, constipation, and obstruction -Biopsy: absent ganglion cells is diagnostic

21 years woman -Abdominal cramps, bloody diarrhea (5 days per day); after egg sandwich -Stool culture: gram negative rods that produce hydrogen sulfide and do not ferment lactose -Effects if she receives antibiotics? a) self-limiting systemic inflammatory response b) pruritic maculopapular rash on the extensor surface c) thrombocytopenia and hemolytic anemia d) orange discoloration of bodily fluids e) prolonged fecal excretion of the pathogen

answer: prolonged fecal excretion of the pathogen -Salmonella gastroenteritis -Treatment: fluoroquinolones (e.g., ciprofloxacin) -Side effect: prolonged fecal excretion of salmonella (use antibiotics only if clinically indicated and for an adequate duration; only for severe cases/diarrhea more than 9 times per day or presence of systemic symptoms (fever, tachycardia, hypotonia) -This patient normal vital signs and 5 times per day diarrhea= doesn't qualify for antibiotics option A= describes Jarisch-Herxheimer reaction, reaction to bacterial endotoxins after antibiotic therapy initiation; after treatment of spirochetes (syphilis (primary skin lesions, regional nontender lymphadenopathy), Lyme disease (erythema chronicum migrans, migratory arthritis, cranial nerve palsy, polyneuropathy and carditis)) option B= infectious mononucleosis; when incorrectly prescribed ampicillin; no signs of IM (splenomegaly, pharyngitis, bilateral cervical lymphadenopathy option C= hemolytic uremic syndrome; sometimes when shigella or EHEC infection treated with antibiotics; E.coli ferments lactose; Shigella doesn't produce hydrogen sulfide option D= side effect of rifampin treatment; treatment for TB (acid fast bacilli; dyspnea, productive cough, weight loss, night sweats, lymphadenopathy) or prophylaxis when in contact with N. meningitidis

15 years boy -RLQ pain, vomiting, abdominal distention -Palpable mass in RLQ -X-ray= dilated ascending colon with air fluid level in small intestine -Test in which electrodes are placed on the nasal epithelium and the nose is perfused with different solutions -Chloride-free solution is administered= hyperpolarization across nasal epithelium is absent -Most common cause of mortality in this patient? a)Liver cirrhosis b)Pulmonary embolism c)Diabetes mellitus d)Pulmonary infection e)Nephrolithiasis

answer: pulmonary infections -Nasal potential difference test= cystic fibrosis -Hyperviscous exocrine gland secretions (thicken stool and cause distal intestinal obstruction) -Hyperviscous respiratory secretions and deficient mucociliary clearance= recurrent pulmonary infections -This combined with multiple episodes of antibiotic therapy= more severe, resistant infections (e.g., pseudomonas aeruginosa) and structural lung changes (e.g., bronchiectasis, emphysema) with eventual progression to respiratory failure

age: 37 years man fatigue, jaundice (3 days) mild RUQ tenderness serum parameters (over 4 months): 1st month: elevated HBsAg, HBeAg and elevated Anti-HBc IgM which then fall 4th month: elevated Anti-HBc IgG, elevated Anti-HBs and anti-HBe explanation? a) Chronic hepatitis B infection with low infectivity b) Chronic hepatitis B infection with high infectivity c) Adverse reaction to hepatitis B infection d) Acute exacerbation of previous hepatitis B infection e) Resolved acute hepatitis B infection

answer: resolved acute hepatitis B infection -Elevated anti-HBc antibodies (IgG) -HBeAg and HBsAg are negative and positive anti-HB and anti-HBs antibodies --> resolved acute infection option A= HBsAg, anti-HBe, anti-HBc IgG option B= HBsAg, HBeAg, anti-HBc IgG option C= vaccination= elevated ONLY anti-HBs antibodies; adverse reactions: redness, swelling, myalgia, nausea, fatigue option D= positive HBsAg, HBeAg, anti-HBc IgM antibodies first but then present with positive anti-HBe antibodies and higher concentration of anti-HBc IgG antibodies

49 years man 6-month history: fatigue, reduced libido Joint pain (3rd metacarpophalangeal joints); tanned skin and small testes Liver is palpated 2-3cm below costal margin Histology: intracellular material that stains with Prussian blue Complication? a) colorectal carcinoma b) restrictive cardiomyopathy c) rheumatoid arthritis d) pancreatic adenocarcinoma e) non-hodgkin lymphoma

answer: restrictive cardiomyopathy -Untreated or advanced hemochromatosis= deposition of iron in cardiac tissue and subsequent fibrosis= 1) restrictive cardiomyopathy (more typical) 2) dilated cardiomyopathy (less common, often reversible) option A= risk factors: FAP and HNPCC, IBD, smoking, low fiber diet option C= risk factors: female, HLA-DR4, smoking; hemochromatosis doesn't cause RA, but can cause calcium pyrophosphate deposition disease and irreversible arthropathy, especially affecting MCP joints option D= risk factors: genetic syndromes (Peutz-Jeghers syndrome), smoking, chronic pancreatitis, diabetes option E= risk factors: infections (EBV, HCV, H. pylori), autoimmune diseases (Hashimoto's), dermatomyositis

10 days newborn -Nonbilious vomiting; stool: soft and yellow colored -Pregnancy : complicated by polyhydramnios -Down syndrome -Mild abdominal distention and normal bowel sounds -X-ray shown -Cause of defect? a) rotation of ventral pancreatic bud b) foregut septation c) ganglion cell migration d) duodenal recanalization e) umbilical ring closure

answer: rotation of ventral pancreatic bud -Embryology (pancreas): endoderm forms two buds (ventral and dorsal) -Foregut rotation= ventral bud fuses with dorsal bud -Defect in fusion= ring of pancreatic tissue surrounding duodenum (annular pancreas) and duodenal stenosis -Antenatal period: polyhydramnios -Neonatal period: feeding intolerance, vomiting (non-bilious) but can be bilious depending on obstruction (proximal or distal duodenum) and abdominal distention -Annular pancreas is associated with down syndrome option B= defect in lateral septation of foregut into esophagus and trachea= esophageal atresia; leads to polyhydramnios, bilious vomiting but also excessive drooling, choking, and respiratory distress; X-ray: airless abdomen not double bubble sign option C= Hirschsprung disease; also associated with down syndrome; bilious vomiting and failure to pass meconium; no double bubble sign option D= duodenal atresia; associated with down syndrome; double bubble sign; but bilious vomiting; delayed passage of meconium seen option E= umbilical hernia; associated with down syndrome; bilious vomiting; abdominal wall defect especially when crying due to increased intraabdominal pressure

47 years woman -Abdominal distention; alcohol daily -Pallor, scleral icterus, fluid wave with shifting dullness -Intravascular pressure increased in? a) short gastric vein b) splenic artery c) inferior epigastric vein d) azygos vein e) gastroduodenal artery

answer: short gastric vein -Alcohol cirrhosis with hypertension -Short gastric veins drain into splenic vein, which joins SMV to form hepatic portal vein -Results in gastric varices option B= branch of celiac, supplies spleen, pancreas as well as fundus and upper greater curvature of stomach option C= drains into external iliac vein, part of systemic circulation; increased blood flow in IEV can occur in portal hypertension as a result of portosystemic shunting via paraumbilical veins and result in caput medusae; less likely than short gastric veins or later option D= drains into SVC; can occur in portal hypertension due to portosystemic shunting; less likely option E= branch of common hepatic artery; pylorus, stomach, duodenum, and head of pancreas (via superior pancreaticoduodenal arteries

38 years woman -History: 2 spontaneous abortions at 11 and 12 weeks of gestation -US= normal liver parenchyma, dilated portal vein and splenic enlargement -Endoscopy= dilated submucosal veins in lower esophagus -Further evaluation shows? a) increased prothrombin time b) palmar erythema c) hepatic venous congestion d) increased serum bilirubin levels e) councilman bodies f) thrombocytopenia

answer: thrombocytopenia -Dilated portal vein + splenomegaly + esophageal varices + normal liver parenchyma= portal hypertension due to portal vein thrombosis -Common presentation in splenomegaly (can increase splenic activity= hypersplenism), leading to rapid clearance of platelets from bloodstream -Splenomegaly and esophageal varices are seen in all forms of hypertension (unlike others, portal vein thrombosis are not associated with ascites) -Repeated miscarriages + portal vein thrombosis= suspect antiphospholipid syndrome option A= in cirrhosis; ascites and features of cirrhosis (increased surface nodularity, coarse or heterogenous echotexture) on US are absent; repeated spontaneous miscarriages + current portal vein thrombosis= suggest antiphospholipid syndrome, PTT is prolonged while PT is normal in APS option B= in cirrhosis, due to hyperestrogenism option C= Budd Chiari syndrome; absence of ascites, hepatomegaly, hypertonic caudate lobe, reversed flow in hepatic veins option D= in cirrhosis option E= acute viral hepatitis; fulminant hepatic failure can result in abdominal pain and rarely portal hypertension; liver is normal here

79 years man -Aortic stenosis; fatigue (5 months) -Intermittent bright red blood stool -No pain or weight loss -Pale conjunctivae and crescendo-decrescendo systolic murmur (2nd right intercostal space) -Low hemoglobin and MCV; colonoscopy normal -Underlying mechanism? a)Thrombus in the superior mesenteric artery b)Transmural inflammation of large bowel c)Atherosclerotic narrowing of the mesenteric arteries d)Tortuous submucosal blood vessels e)Inherited factor VIII deficiency

answer: tortuous submucosal blood vessels -Tortuous and dilated submucosal blood vessels= angiodysplasia -Associated with aortic stenosis -Dysplastic vessels typically located in cecum and ascending colon (75%) -Episodic and self-limiting bleeding -Diagnosis: angiography -Other conditions associated with angiodysplasia: vWD and end-stage renal disease option A= acute mesenteric ischemia; thromboembolism of mesenteric arteries; acute severe abdominal pain followed by currant jelly bloody stools due to bowel necrosis; intermittent bleeding and pain usually occurs option B= crohn disease option C= chronic mesenteric ischemia; postprandial pain; abdominal angina; weight loss; no hematochezia option E= hemophilia A; can cause GI bleeding; but also, spontaneous and prolonged epistaxis, hemarthrosis beginning early in life

19 years woman -Abdominal pain, bloating, episodic diarrhea; weight loss -Mildly distended abdomen, diffuse abdominal tenderness -Multiple erythematous tender nodules (anterior legs) -Draining lesion in perianal region -Biopsy? a) villous atrophy b) crypt abscesses c) no structural abnormalities d) melanosis coli e) neuroendocrine tumor cells f) transmural inflammation

answer: transmural inflammation -Erythema nodosum + perianal fistula= Crohn disease -Transmural inflammation; cobblestone appearance; discontinuous linear ulcer; noncaseating granuloma option A= celiac option B= ulcerative colitis option C= irritable bowel syndrome or lactose intolerance option D= benign hyperpigmentation of colonic epithelium in patients who abuse laxatives; abdominal pain, diarrhea, weight loss option E= carcinoid tumors

34 years man -Palpitation, shortness of breath, diarrhea, abdominal cramps (2 months) -Cutaneous flushing; bilateral wheezing -Urine collection: increased 5-HIAA -CT= intestinal tumor with extensive metastasis to liver -Diagnosis: inoperable, treatment started with octreotide -Abdominal pain and frequent loose stools continued after treatment -Suggest trial to test addition treatment of new drug that can improve system -New drug inhibits? a)Dopamine beta-hydroxylase b)Vasoactive intestinal peptide c)Plasma kallikrein d)Histidine decarboxylase e)Tryptophan hydroxylase

answer: tryptophan hydroxylase -Symptoms and findings of carcinoid tumor -Tryptophan hydroxylase à converts tryptophan to serotonin -Carcinoid tumor: excess production of serotonin (responsible for GI symptoms) -Telotristat= inhibitor of tryptophan hydroxylase (approved for patients with carcinoid tumors with persistent diarrhea (not controlled by somatostatin analogs (octreotide) option A= dopamine --> norepinephrine; used in pheochromocytoma option B= VIPomas option C= kininogen --> bradykinin; bradykinin (cough, dyspnea, abdominal pain, diarrhea); carcinoid tumors have high kallikrein (but kallikrein inhibitors like ecallantide are not approved in treatment); they treat bradykinin excess in hereditary angioedema option D= histidine --> histamine; can cause dyspnea, wheezing, cutaneous flushing, abdominal cramps and diarrhea; carcinoid tumors produce histamine; potentially beneficial but not approved for clinical use

67 years woman -Progressive fatigue, pallor; low hemoglobin and MCV and serum ferritin -Occult blood positive -Colonoscopy: irregular, bleeding 3-cm exophytic ulcer in right colon -Greatest risk factor for this condition? a) submucosal lipomatous polyp b) sessile hamartomatous polyp c) serrated hyperplastic polyp d) villous adenomatous polyp e) tubular adenomatous polyp f) submucosal leiomyomatous polyp g) pedunculated inflammatory polyp

answer: villous adenomatous polyp -Symptomatic iron deficiency anemia, irregular ulcerative growth= colorectal cancer -Villous adenomatous polyps carry highest risk of malignant transformation (50%) option A= very low risk of malignant transformation; lipomas are most commonly subcutaneously; when in colon, they can become large, fungating and ulcerated= mimic primary colorectal cancer; but do not cause symptoms patient has option B= benign; when associated with syndromes, higher risk option C= 3 types of serrated= hyperplastic, sessile serrated, and traditional serrated); moderate risk; hyperplastic has lowest risk of the 3= no dysplasia present option E= small risk (5%) option G= seen with IBD; polyp itself has no risk; while tissue surrounding it has higher risk of neoplastic transformation

69 years man -Difficulty swallowing (solids then liquids too), weight loss (3 months) -Large mass 3cm proximal to esophagogastric junction -Biopsy: distortion of glandular architecture -Strongest risk factor? a) atrophic gastritis b) consumption of hot liquids c) chronic alcohol use d) visceral obesity e) chewing of betel nuts f) consumption of cured meats

answer: visceral obesity -Histology + location= esophageal adenocarcinoma -Hypothesis --> obesity= increased risk of GERD= metaplasia= adenocarcinoma -Excessive visceral fat= risk of inflammation, production of cytokines, and insulin resistance= risk factors for carcinoma all other options= eSCC


संबंधित स्टडी सेट्स

Chapter 21: Respiratory Care Modalities

View Set

Pediatric HESI Practice Questions

View Set

Dual Credit English IV Final Review (I finessed)

View Set

Week 14 - Chapter 16: Nursing Management During the Postpartum Period

View Set

Auditing & assurance services ch 5

View Set